What are some noteworthy “mic-drop” moments in math?Philosophy behind Yitang Zhang's work on the Twin Primes ConjectureHow did Cole factor $2^67-1$ in 1903?With what equation could aliens prove their superior intelligence?What are some slogans that express mathematical tricks?What are some good resources for mathematical translation?What are some famous rejections of correct mathematics?What are some applications of other fields to mathematics?What are some good group theory references?What are some mathematical sculptures?Downsides of using the arXiv? What are some Applications of Teichmüller Theory?What are some deep theorems, and why are they considered deep?Higher Moments, what are they good for?

What are some noteworthy “mic-drop” moments in math?


Philosophy behind Yitang Zhang's work on the Twin Primes ConjectureHow did Cole factor $2^67-1$ in 1903?With what equation could aliens prove their superior intelligence?What are some slogans that express mathematical tricks?What are some good resources for mathematical translation?What are some famous rejections of correct mathematics?What are some applications of other fields to mathematics?What are some good group theory references?What are some mathematical sculptures?Downsides of using the arXiv? What are some Applications of Teichmüller Theory?What are some deep theorems, and why are they considered deep?Higher Moments, what are they good for?













85












$begingroup$


Oftentimes in math the manner in which a solution to a problem is announced becomes a significant chapter/part of the lore associated with the problem, almost being remembered more than the manner in which the problem was solved. I think that most mathematicians as a whole, even upon solving major open problems, are an extremely humble lot. But as an outsider I appreciate the understated manner in which some results are dropped.



The very recent example that inspired this question:



  • Andrew Booker's recent solution to $a^3+b^3+c^3=33$ with $(a,b,c)inmathbbZ^3$ as $$(a,b,c)=(8866128975287528,-8778405442862239,-2736111468807040)$$ was publicized on Tim Browning's homepage. However the homepage has merely a single, austere line, and does not even indicate that this is/was a semi-famous open problem. Nor was there any indication that the cubes actually sum to $33$, apparently leaving it as an exercise for the reader.

Other examples that come to mind include:



  • In 1976 after Appel and Hakken had proved the Four Color Theorem, Appel wrote on the University of Illinois' math department blackboard "Modulo careful checking, it appears that four colors suffice." The statement "Four Colors Suffice" was used as the stamp for the University of Illinois at least around 1976.

  • In 1697 Newton famously offered an "anonymous solution" to the Royal Society to the Brachistochrone problem that took him a mere evening/sleepless night to resolve. I think the story is noteworthy also because Johanne Bernoulli is said "recognized the lion by his paw."

  • As close to a literal "mic-drop" as I can think of, after noting in his 1993 lectures that Fermat's Last Theorem was a mere corollary of the work presented, Andrew Wiles famously ended his lecture by stating "I think I'll stop here."


What are other noteworthy examples of such announcements in math that are, in some sense, memorable for being understated? Say to an outsider in the field?




Watson and Crick's famous ending of their DNA paper, "It has not escaped our notice that the specific pairing we have postulated immediately suggests a possible copying mechanism for the genetic material," has a bit of the same understated feel...










share|cite|improve this question











$endgroup$







  • 11




    $begingroup$
    The tale about Cole seems to have no basis in fact and was just a legend propagated by E. T. Bell, who was a former PhD student of Cole. Cole did have a real method of discovering the factorization (the answers to mathoverflow.net/questions/207321/… include a link to Cole's article) and it was not the "three years of Sundays" that Bell wrote. I therefore don't think the Cole story should be among your examples.
    $endgroup$
    – KConrad
    Mar 10 at 20:37







  • 6




    $begingroup$
    The example of how Ramanujan's results came to the attention of Hardy and Littlewood is fairly well documented, and would be a better choice than Cole's "story".
    $endgroup$
    – KConrad
    Mar 10 at 20:44







  • 7




    $begingroup$
    Tim Browning announced the three-cubes solution, but it seems that he was reporting on work of Andrew Booker, see gilkalai.wordpress.com/2019/03/09/… and people.maths.bris.ac.uk/~maarb/papers/cubesv1.pdf
    $endgroup$
    – Gerry Myerson
    Mar 10 at 22:02






  • 8




    $begingroup$
    This is perhaps an example of the opposite phenomenon. It's from my own hazy memory, and I don't fully recall the particulars, including how I happened to overhear this. (Maybe I was just walking by at the right time?). But one morning while I was in grad school at Chicago in the 1970s, Yitz Herstein walked into Irving Kaplansky's office and announced that "Last night, I proved a beautiful theorem". To which Kaplanskky replied: "Wouldn't it have been better if you'd waited for me to say that?"
    $endgroup$
    – Steven Landsburg
    Mar 11 at 1:48






  • 7




    $begingroup$
    Even soft questions deserve accurate answers....the answers here are an ahistorical embarassment.
    $endgroup$
    – Matt F.
    2 days ago















85












$begingroup$


Oftentimes in math the manner in which a solution to a problem is announced becomes a significant chapter/part of the lore associated with the problem, almost being remembered more than the manner in which the problem was solved. I think that most mathematicians as a whole, even upon solving major open problems, are an extremely humble lot. But as an outsider I appreciate the understated manner in which some results are dropped.



The very recent example that inspired this question:



  • Andrew Booker's recent solution to $a^3+b^3+c^3=33$ with $(a,b,c)inmathbbZ^3$ as $$(a,b,c)=(8866128975287528,-8778405442862239,-2736111468807040)$$ was publicized on Tim Browning's homepage. However the homepage has merely a single, austere line, and does not even indicate that this is/was a semi-famous open problem. Nor was there any indication that the cubes actually sum to $33$, apparently leaving it as an exercise for the reader.

Other examples that come to mind include:



  • In 1976 after Appel and Hakken had proved the Four Color Theorem, Appel wrote on the University of Illinois' math department blackboard "Modulo careful checking, it appears that four colors suffice." The statement "Four Colors Suffice" was used as the stamp for the University of Illinois at least around 1976.

  • In 1697 Newton famously offered an "anonymous solution" to the Royal Society to the Brachistochrone problem that took him a mere evening/sleepless night to resolve. I think the story is noteworthy also because Johanne Bernoulli is said "recognized the lion by his paw."

  • As close to a literal "mic-drop" as I can think of, after noting in his 1993 lectures that Fermat's Last Theorem was a mere corollary of the work presented, Andrew Wiles famously ended his lecture by stating "I think I'll stop here."


What are other noteworthy examples of such announcements in math that are, in some sense, memorable for being understated? Say to an outsider in the field?




Watson and Crick's famous ending of their DNA paper, "It has not escaped our notice that the specific pairing we have postulated immediately suggests a possible copying mechanism for the genetic material," has a bit of the same understated feel...










share|cite|improve this question











$endgroup$







  • 11




    $begingroup$
    The tale about Cole seems to have no basis in fact and was just a legend propagated by E. T. Bell, who was a former PhD student of Cole. Cole did have a real method of discovering the factorization (the answers to mathoverflow.net/questions/207321/… include a link to Cole's article) and it was not the "three years of Sundays" that Bell wrote. I therefore don't think the Cole story should be among your examples.
    $endgroup$
    – KConrad
    Mar 10 at 20:37







  • 6




    $begingroup$
    The example of how Ramanujan's results came to the attention of Hardy and Littlewood is fairly well documented, and would be a better choice than Cole's "story".
    $endgroup$
    – KConrad
    Mar 10 at 20:44







  • 7




    $begingroup$
    Tim Browning announced the three-cubes solution, but it seems that he was reporting on work of Andrew Booker, see gilkalai.wordpress.com/2019/03/09/… and people.maths.bris.ac.uk/~maarb/papers/cubesv1.pdf
    $endgroup$
    – Gerry Myerson
    Mar 10 at 22:02






  • 8




    $begingroup$
    This is perhaps an example of the opposite phenomenon. It's from my own hazy memory, and I don't fully recall the particulars, including how I happened to overhear this. (Maybe I was just walking by at the right time?). But one morning while I was in grad school at Chicago in the 1970s, Yitz Herstein walked into Irving Kaplansky's office and announced that "Last night, I proved a beautiful theorem". To which Kaplanskky replied: "Wouldn't it have been better if you'd waited for me to say that?"
    $endgroup$
    – Steven Landsburg
    Mar 11 at 1:48






  • 7




    $begingroup$
    Even soft questions deserve accurate answers....the answers here are an ahistorical embarassment.
    $endgroup$
    – Matt F.
    2 days ago













85












85








85


35



$begingroup$


Oftentimes in math the manner in which a solution to a problem is announced becomes a significant chapter/part of the lore associated with the problem, almost being remembered more than the manner in which the problem was solved. I think that most mathematicians as a whole, even upon solving major open problems, are an extremely humble lot. But as an outsider I appreciate the understated manner in which some results are dropped.



The very recent example that inspired this question:



  • Andrew Booker's recent solution to $a^3+b^3+c^3=33$ with $(a,b,c)inmathbbZ^3$ as $$(a,b,c)=(8866128975287528,-8778405442862239,-2736111468807040)$$ was publicized on Tim Browning's homepage. However the homepage has merely a single, austere line, and does not even indicate that this is/was a semi-famous open problem. Nor was there any indication that the cubes actually sum to $33$, apparently leaving it as an exercise for the reader.

Other examples that come to mind include:



  • In 1976 after Appel and Hakken had proved the Four Color Theorem, Appel wrote on the University of Illinois' math department blackboard "Modulo careful checking, it appears that four colors suffice." The statement "Four Colors Suffice" was used as the stamp for the University of Illinois at least around 1976.

  • In 1697 Newton famously offered an "anonymous solution" to the Royal Society to the Brachistochrone problem that took him a mere evening/sleepless night to resolve. I think the story is noteworthy also because Johanne Bernoulli is said "recognized the lion by his paw."

  • As close to a literal "mic-drop" as I can think of, after noting in his 1993 lectures that Fermat's Last Theorem was a mere corollary of the work presented, Andrew Wiles famously ended his lecture by stating "I think I'll stop here."


What are other noteworthy examples of such announcements in math that are, in some sense, memorable for being understated? Say to an outsider in the field?




Watson and Crick's famous ending of their DNA paper, "It has not escaped our notice that the specific pairing we have postulated immediately suggests a possible copying mechanism for the genetic material," has a bit of the same understated feel...










share|cite|improve this question











$endgroup$




Oftentimes in math the manner in which a solution to a problem is announced becomes a significant chapter/part of the lore associated with the problem, almost being remembered more than the manner in which the problem was solved. I think that most mathematicians as a whole, even upon solving major open problems, are an extremely humble lot. But as an outsider I appreciate the understated manner in which some results are dropped.



The very recent example that inspired this question:



  • Andrew Booker's recent solution to $a^3+b^3+c^3=33$ with $(a,b,c)inmathbbZ^3$ as $$(a,b,c)=(8866128975287528,-8778405442862239,-2736111468807040)$$ was publicized on Tim Browning's homepage. However the homepage has merely a single, austere line, and does not even indicate that this is/was a semi-famous open problem. Nor was there any indication that the cubes actually sum to $33$, apparently leaving it as an exercise for the reader.

Other examples that come to mind include:



  • In 1976 after Appel and Hakken had proved the Four Color Theorem, Appel wrote on the University of Illinois' math department blackboard "Modulo careful checking, it appears that four colors suffice." The statement "Four Colors Suffice" was used as the stamp for the University of Illinois at least around 1976.

  • In 1697 Newton famously offered an "anonymous solution" to the Royal Society to the Brachistochrone problem that took him a mere evening/sleepless night to resolve. I think the story is noteworthy also because Johanne Bernoulli is said "recognized the lion by his paw."

  • As close to a literal "mic-drop" as I can think of, after noting in his 1993 lectures that Fermat's Last Theorem was a mere corollary of the work presented, Andrew Wiles famously ended his lecture by stating "I think I'll stop here."


What are other noteworthy examples of such announcements in math that are, in some sense, memorable for being understated? Say to an outsider in the field?




Watson and Crick's famous ending of their DNA paper, "It has not escaped our notice that the specific pairing we have postulated immediately suggests a possible copying mechanism for the genetic material," has a bit of the same understated feel...







soft-question big-list






share|cite|improve this question















share|cite|improve this question













share|cite|improve this question




share|cite|improve this question








edited 2 days ago


























community wiki





9 revs, 3 users 95%
Mark S








  • 11




    $begingroup$
    The tale about Cole seems to have no basis in fact and was just a legend propagated by E. T. Bell, who was a former PhD student of Cole. Cole did have a real method of discovering the factorization (the answers to mathoverflow.net/questions/207321/… include a link to Cole's article) and it was not the "three years of Sundays" that Bell wrote. I therefore don't think the Cole story should be among your examples.
    $endgroup$
    – KConrad
    Mar 10 at 20:37







  • 6




    $begingroup$
    The example of how Ramanujan's results came to the attention of Hardy and Littlewood is fairly well documented, and would be a better choice than Cole's "story".
    $endgroup$
    – KConrad
    Mar 10 at 20:44







  • 7




    $begingroup$
    Tim Browning announced the three-cubes solution, but it seems that he was reporting on work of Andrew Booker, see gilkalai.wordpress.com/2019/03/09/… and people.maths.bris.ac.uk/~maarb/papers/cubesv1.pdf
    $endgroup$
    – Gerry Myerson
    Mar 10 at 22:02






  • 8




    $begingroup$
    This is perhaps an example of the opposite phenomenon. It's from my own hazy memory, and I don't fully recall the particulars, including how I happened to overhear this. (Maybe I was just walking by at the right time?). But one morning while I was in grad school at Chicago in the 1970s, Yitz Herstein walked into Irving Kaplansky's office and announced that "Last night, I proved a beautiful theorem". To which Kaplanskky replied: "Wouldn't it have been better if you'd waited for me to say that?"
    $endgroup$
    – Steven Landsburg
    Mar 11 at 1:48






  • 7




    $begingroup$
    Even soft questions deserve accurate answers....the answers here are an ahistorical embarassment.
    $endgroup$
    – Matt F.
    2 days ago












  • 11




    $begingroup$
    The tale about Cole seems to have no basis in fact and was just a legend propagated by E. T. Bell, who was a former PhD student of Cole. Cole did have a real method of discovering the factorization (the answers to mathoverflow.net/questions/207321/… include a link to Cole's article) and it was not the "three years of Sundays" that Bell wrote. I therefore don't think the Cole story should be among your examples.
    $endgroup$
    – KConrad
    Mar 10 at 20:37







  • 6




    $begingroup$
    The example of how Ramanujan's results came to the attention of Hardy and Littlewood is fairly well documented, and would be a better choice than Cole's "story".
    $endgroup$
    – KConrad
    Mar 10 at 20:44







  • 7




    $begingroup$
    Tim Browning announced the three-cubes solution, but it seems that he was reporting on work of Andrew Booker, see gilkalai.wordpress.com/2019/03/09/… and people.maths.bris.ac.uk/~maarb/papers/cubesv1.pdf
    $endgroup$
    – Gerry Myerson
    Mar 10 at 22:02






  • 8




    $begingroup$
    This is perhaps an example of the opposite phenomenon. It's from my own hazy memory, and I don't fully recall the particulars, including how I happened to overhear this. (Maybe I was just walking by at the right time?). But one morning while I was in grad school at Chicago in the 1970s, Yitz Herstein walked into Irving Kaplansky's office and announced that "Last night, I proved a beautiful theorem". To which Kaplanskky replied: "Wouldn't it have been better if you'd waited for me to say that?"
    $endgroup$
    – Steven Landsburg
    Mar 11 at 1:48






  • 7




    $begingroup$
    Even soft questions deserve accurate answers....the answers here are an ahistorical embarassment.
    $endgroup$
    – Matt F.
    2 days ago







11




11




$begingroup$
The tale about Cole seems to have no basis in fact and was just a legend propagated by E. T. Bell, who was a former PhD student of Cole. Cole did have a real method of discovering the factorization (the answers to mathoverflow.net/questions/207321/… include a link to Cole's article) and it was not the "three years of Sundays" that Bell wrote. I therefore don't think the Cole story should be among your examples.
$endgroup$
– KConrad
Mar 10 at 20:37





$begingroup$
The tale about Cole seems to have no basis in fact and was just a legend propagated by E. T. Bell, who was a former PhD student of Cole. Cole did have a real method of discovering the factorization (the answers to mathoverflow.net/questions/207321/… include a link to Cole's article) and it was not the "three years of Sundays" that Bell wrote. I therefore don't think the Cole story should be among your examples.
$endgroup$
– KConrad
Mar 10 at 20:37





6




6




$begingroup$
The example of how Ramanujan's results came to the attention of Hardy and Littlewood is fairly well documented, and would be a better choice than Cole's "story".
$endgroup$
– KConrad
Mar 10 at 20:44





$begingroup$
The example of how Ramanujan's results came to the attention of Hardy and Littlewood is fairly well documented, and would be a better choice than Cole's "story".
$endgroup$
– KConrad
Mar 10 at 20:44





7




7




$begingroup$
Tim Browning announced the three-cubes solution, but it seems that he was reporting on work of Andrew Booker, see gilkalai.wordpress.com/2019/03/09/… and people.maths.bris.ac.uk/~maarb/papers/cubesv1.pdf
$endgroup$
– Gerry Myerson
Mar 10 at 22:02




$begingroup$
Tim Browning announced the three-cubes solution, but it seems that he was reporting on work of Andrew Booker, see gilkalai.wordpress.com/2019/03/09/… and people.maths.bris.ac.uk/~maarb/papers/cubesv1.pdf
$endgroup$
– Gerry Myerson
Mar 10 at 22:02




8




8




$begingroup$
This is perhaps an example of the opposite phenomenon. It's from my own hazy memory, and I don't fully recall the particulars, including how I happened to overhear this. (Maybe I was just walking by at the right time?). But one morning while I was in grad school at Chicago in the 1970s, Yitz Herstein walked into Irving Kaplansky's office and announced that "Last night, I proved a beautiful theorem". To which Kaplanskky replied: "Wouldn't it have been better if you'd waited for me to say that?"
$endgroup$
– Steven Landsburg
Mar 11 at 1:48




$begingroup$
This is perhaps an example of the opposite phenomenon. It's from my own hazy memory, and I don't fully recall the particulars, including how I happened to overhear this. (Maybe I was just walking by at the right time?). But one morning while I was in grad school at Chicago in the 1970s, Yitz Herstein walked into Irving Kaplansky's office and announced that "Last night, I proved a beautiful theorem". To which Kaplanskky replied: "Wouldn't it have been better if you'd waited for me to say that?"
$endgroup$
– Steven Landsburg
Mar 11 at 1:48




7




7




$begingroup$
Even soft questions deserve accurate answers....the answers here are an ahistorical embarassment.
$endgroup$
– Matt F.
2 days ago




$begingroup$
Even soft questions deserve accurate answers....the answers here are an ahistorical embarassment.
$endgroup$
– Matt F.
2 days ago










11 Answers
11






active

oldest

votes


















69












$begingroup$

The best known lower bound for the minimal length of superpermutations was originally posted anonymously to 4chan.



The story is told at Mystery Math Whiz and Novelist Advance Permutation Problem, and a publication with a cleaned-up version of the proof is at A lower bound on the length of the shortest superpattern, with "Anonymous 4chan Poster" as the first author. The original 4chan source is archived here.






share|cite|improve this answer











$endgroup$








  • 15




    $begingroup$
    Also: a new superpermutation of 7 symbols, shorter than any that was known at the time (8907 symbols long), was posted as a pseudonymous comment on YouTube in February 2019.
    $endgroup$
    – Robin Houston
    Mar 10 at 22:05







  • 24




    $begingroup$
    "Mainly devoted to anime" is a rather kind way to put it.. ;-)
    $endgroup$
    – R..
    Mar 10 at 22:29






  • 18




    $begingroup$
    @R.., as I understand it, the 4chan poster answered a question in a forum dedicated to a particular anime program. The anime in question was meant to be non-linear, and watched in any order. The question was effectively "what is the most efficient way to watch all $n$ episodes of the anime serially, in any order." So it was answered in a forum really "devoted to anime," rather than the average 4chan forum.
    $endgroup$
    – Mark S
    Mar 10 at 23:32







  • 6




    $begingroup$
    @MarkS More specifically, Suzumiya Haruhi no Yuuutsu, and hence the problem was also named "The Haruhi Problem".
    $endgroup$
    – Pedro A
    Mar 11 at 2:04






  • 7




    $begingroup$
    It’s far too late for me to edit my first comment, but I should say that the superpermutation posted as a YouTube comment is 5907 symbols long, not 8907. Apologies for mistyping, and for not noticing sooner.
    $endgroup$
    – Robin Houston
    yesterday


















44












$begingroup$

I'll just let below (famous) 1966 article from the Bulletin of the American Mathematical Society speak for itself...



Lander and Parkin, BAMS 1966




COUNTEREXAMPLE TO EULER'S CONJECTURE ON SUMS OF LIKE POWERS

BY L. J. LANDER AND T. R. PARKIN

Communicated by J. D. Swift, June 27, 1966



A direct search on the CDC 6600 yielded



$$ 27^5 + 84^5 + 110^5 + 133^5 = 144^5 $$



as the smallest instance in which four fifth powers sum to a fifth power. This is a counterexample to a conjecture by Euler [1] that at least $ n $ $ n $th powers are required to sum to an $ n $th power, $ n > 2 $.



REFERENCE

1. L. E. Dickson, History of the theory of numbers, Vol. 2, Chelsea, New York, 1952, p. 648.







share|cite|improve this answer











$endgroup$




















    40












    $begingroup$

    Perelman solving the Poincare "conjecture," posting it only on the arXiv, leaving math, and refusing the Clay prize could be interpreted as a kind of "mic drop."






    share|cite|improve this answer











    $endgroup$








    • 13




      $begingroup$
      Let us not mince words: " 'I'm not interested in money or fame,' he is quoted to have said at the time. 'I don't want to be on display like an animal in a zoo. I'm not a hero of mathematics. I'm not even that successful; that is why I don't want to have everybody looking at me.' "
      $endgroup$
      – Samantha Y
      Mar 11 at 2:40






    • 6




      $begingroup$
      The announcement was posting on arxiv (with no buzzword). Refusing prizes occurred several years later hence is not part of the announcement.
      $endgroup$
      – YCor
      2 days ago






    • 7




      $begingroup$
      @SamanthaY Ironically, I think he's succumbed to the Streisand Effect by doing that. A significant fraction of my awareness of Perelman and the results he's credited with is a result of his refusals to be recognized.
      $endgroup$
      – zibadawa timmy
      2 days ago






    • 6




      $begingroup$
      @zibadawatimmy Yes, but Perelman refused to be owned by the public, which is quite a different thing than wanting privacy/anonymity. To that end, I feel he succeeded.
      $endgroup$
      – Samantha Y
      2 days ago






    • 4




      $begingroup$
      @ycor I apologize :( this entire thread strikes me as unproductive, but that doesn't excuse my taking offense at you, especially as we seem to be in agreement. That is (if I may say so), perpetuating mythologizing the Perelman story, or characterising it as a 'mic drop' will only drive a wedge between mathematicians and the general public, by portraying us as either prize-driven or "odd-balls", and even if somewhere between these two, as drama obsessed gossips. After all, (and this is a very general comment) don't we want everyone to be mathematician? Let them see we're human.
      $endgroup$
      – Samantha Y
      2 days ago



















    29












    $begingroup$

    Not math but in physics the statistical interpretation of the wave-function was announced by Max Born in a footnote.



    From his paper Zur Quantenmechanik der Stoßvorgänge,




    (1) Anmerkung bei der Korrektur: Genauere Überlegung zeigt, daß die
    Wahrscheinlichkeit dem Quadrat der Größe $Phi_n_tau m$ proportional ist.




    This can be translated as




    (1) Addition in proof: More careful consideration shows that the probability is proportional to the square
    of the quantity $Phi_n_tau m.$




    Because of its implications this is probably the most important footnote in the history of physics. Max Born was awarded the Nobel prize "for his fundamental research in quantum mechanics, especially for his statistical interpretation of the wavefunction".






    share|cite|improve this answer











    $endgroup$








    • 5




      $begingroup$
      I think "Anmerkung bei der Korrektur" is better translated as "Remark added in proof". In particular, it would be a remark by the author, not by the editor. Also, "zeigt" is present tense, "shows" not "will show".
      $endgroup$
      – Andreas Blass
      Mar 11 at 1:44






    • 14




      $begingroup$
      The footnote is not the announcement of a probabilistic interpretation, but a correction that the probability is proportional to $Phi^2$ rather than $Phi$. Also the paper is not so much understated as preliminary, as indicated right below the title.
      $endgroup$
      – Matt F.
      Mar 11 at 2:41











    • $begingroup$
      @AndreasBlass you're right. You're welcome to provide a better translation than the one I found online. If I remember correctly Born added that footnote once the paper was already in the review process
      $endgroup$
      – lcv
      Mar 11 at 3:14






    • 2




      $begingroup$
      @MattF. I take it that your comment means that he didn't mean to show understatement with the footnote. I totally agree. This is however, how the statistical interpretation was brought to public attention. De facto so to speak.
      $endgroup$
      – lcv
      2 days ago


















    28












    $begingroup$

    Applications of algebra to a problem in topology (YouTube) at Atiyah80 was a talk by Mike Hopkins. In it he announced the solution to the Kervaire invariant one problem in all but one dimension (arXiv, Annals).






    share|cite|improve this answer











    $endgroup$








    • 26




      $begingroup$
      I was in the audience as a second year graduate student, and for me it was just another lecture where I could only understand the first 10-15 minutes. A stranger sitting next to me was very excited afterward, but I remember wondering if he was a nut. So yeah, this qualifies as a mic drop in my book.
      $endgroup$
      – Paul Siegel
      2 days ago










    • $begingroup$
      Can I ask a question about the video? At the beginning there is some discussion of various $pi_n(S^0)$, in particular Pontryagin first errornously claiming that $pi_2(S^0) = 0$ and later realizing that in fact $pi_2(S^0) = mathbbZ/2mathbbZ$. So here is my question: isn't $S^0$ just two points? What would a representative of the non-zero class of $pi_2(S^0)$ look like? You can't get both points of $S^0$ in the range of the map, can you? What is going on here?
      $endgroup$
      – Vincent
      20 hours ago






    • 2




      $begingroup$
      @Vincent from memory he's most likely discussing stable homotopy groups, so that really it's $pi_n+2(S^n)$ for large enough $n$ (in this case, $ngeq 4$), but being sloppy with notation. It would be better denoted $pi_2^s(S^0)$. See eg the pink diagonal in this table starting at $pi_6(S^4)$.
      $endgroup$
      – David Roberts
      20 hours ago







    • 1




      $begingroup$
      @Vincent actually, it's saying the same thing, but I think Hopkins would phrase it as looking at the homotopy groups of the sphere spectrum $mathbbS$, which is cooked up out of $S^0$. Then $pi_2(mathbbS)$ is indeed as he describes.
      $endgroup$
      – David Roberts
      19 hours ago










    • $begingroup$
      @DavidRoberts Thanks!
      $endgroup$
      – Vincent
      19 hours ago


















    23












    $begingroup$

    I consider this manner as a mark of a professional mathematician: let others convey the excitement of a discovery. A good recent example was the submission of a paper on bounded gaps between primes. Much of the public excitement was generated by people other than the author, Yitang Zhang.



    Gerhard "Can Be Excited In Private" Paseman, 2019.03.10.






    share|cite|improve this answer











    $endgroup$








    • 3




      $begingroup$
      I especially like his understated comment that "I believe one could make it sharper" when asked if he thought $k<70,000,000$ could be reduced.
      $endgroup$
      – Mark S
      Mar 10 at 23:17






    • 5




      $begingroup$
      Well, a distinction can be drawn between the most professional approach, which I guess is to submit the work to the Annals or another top journal, accept invitations to speak about it, etc. followed by Yitang Zhang and the more dramatic (and fun) approach where you post it only to your personal website, refuse to tell people what your talk announcing the result is about in advance, leave math immediately afterwards, etc. It seems that the "mic drop" refers to examples that go above and beyond what you'd do for a usual strong result.
      $endgroup$
      – Will Sawin
      Mar 11 at 1:34






    • 1




      $begingroup$
      And then there is of course Grothendieck who let Borel & Serre publish his proof of the GRR theorem...
      $endgroup$
      – Einfacher Schreiberling
      yesterday


















    22












    $begingroup$

    From the Wikipedia article on Frank Nelson Cole:




    On October 31, 1903, Cole famously made a presentation to a meeting of
    the American Mathematical Society where he identified the factors of
    the Mersenne number $2^67$ − 1, or M67.[5] Édouard Lucas had demonstrated
    in 1876 that M67 must have factors (i.e., is not prime), but he was
    unable to determine what those factors were. During Cole's so-called
    "lecture", he approached the chalkboard and in complete silence
    proceeded to calculate the value of M67, with the result being
    147,573,952,589,676,412,927. Cole then moved to the other side of the
    board and wrote 193,707,721 × 761,838,257,287, and worked through the
    tedious calculations by hand. Upon completing the multiplication and
    demonstrating that the result equaled M67, Cole returned to his seat,
    not having uttered a word during the hour-long presentation. His
    audience greeted the presentation with a standing ovation.







    share|cite|improve this answer











    $endgroup$








    • 13




      $begingroup$
      I'm interested in the historiography of this urban legend. Is the only source for the above E. T. Bell? If so, must it be considered suspect, because E. T. Bell was a much better mythmaker than a biographer? I'd like to believe it to be true - a broken clock is still right twice a day...
      $endgroup$
      – Mark S
      Mar 11 at 0:41






    • 12




      $begingroup$
      Maybe things were different in 1903, but I would not give a standing ovation for an hour of silent arithmetic. Also I’m sorry but those calculations don’t seem like they would take an hour. None of it seems believable. Still a fun story though.
      $endgroup$
      – Zach Teitler
      Mar 11 at 1:18






    • 3




      $begingroup$
      @ZachTeitler Maybe $M_67$ was a really big deal in 1903? Maybe actually finding the factors was generally greeted with some expression of acclamation? Mersenne antedates Fermat by a dozen or so years, $M_67$ was effectively open for just as long in 1903 as FLT was. I'm pretty sure that people stood up and clapped at the end of Wiles' lecture in 1993. Of course Wiles' lecture was not an "hour of silent arithmetic," so maybe that part is a stretch.
      $endgroup$
      – Mark S
      Mar 11 at 2:57







    • 5




      $begingroup$
      $M_67$ would be a big deal any time and finding those factors would have certainly been worthy of acclaim. I just meant that there would be far better ways to present the factorization than grinding through the arithmetic. As an audience member I would be far, far more interested in how Cole found those factors, than in whether he remembered to carry the $3$ or whatever. An hour of that would have been tough to sit through. Although... maybe at one of those 20-minute AMS special sessions, perhaps.... :-)
      $endgroup$
      – Zach Teitler
      Mar 11 at 5:32






    • 3




      $begingroup$
      @ZachTeitler right, so the "applause/standing ovation" is not so suspect, but the "silent lecture of multiplication" seems bogus and inconsistent with even mathematicians of 1903...
      $endgroup$
      – Mark S
      2 days ago


















    17












    $begingroup$

    Kurt Gödel, only a few days before Hilbert gives his famous "We must know – We will know!" quote, had just proven that we cannot know.



    Namely, any reasonably strong foundation of mathematics, if it has a finitary proof verification process, cannot decide all the true statements. Mathematics, in its essence, is incomplete.



    Philosophically speaking, perhaps one of the biggest mic drop moments. Metaphorically, this virtual coinciding with Hilbert's lecture just makes the room even more silent afterwards.






    share|cite|improve this answer











    $endgroup$








    • 1




      $begingroup$
      Cool story - when was the lecture? Is there any contemporaneous reporting of the audience reaction?
      $endgroup$
      – Mark S
      2 days ago






    • 18




      $begingroup$
      This took place in 1930 in Koenigsberg, which was Hilbert's hometown. Goedel's talk was a day or two before Hilbert's talk, not at the same conference, and Hilbert may not have even been at that talk. For more, see hsm.stackexchange.com/questions/29/… and maa.org/book/export/html/326610.
      $endgroup$
      – KConrad
      2 days ago











    • $begingroup$
      @KConrad: The silent room is a metaphor here. Because the usual reaction to a mic drop is that the room goes silent for a moment. :-)
      $endgroup$
      – Asaf Karagila
      2 days ago






    • 2




      $begingroup$
      Apparently von Neumann, in the audience, remarked at the end of Gödel's lecture "It's all over". Unfortunately I do not have a good source for this.
      $endgroup$
      – David Roberts
      18 hours ago


















    7












    $begingroup$

    My favorite is non-mathematician Marjorie Rice challenging the proof of "No other pentagon tilings exist" with multiple new pentagon tilings. Schattschneider's article was the primary announcement of the results.






    share|cite|improve this answer











    $endgroup$








    • 6




      $begingroup$
      While an excellent result, how was this a "mic drop"?
      $endgroup$
      – Noah Schweber
      2 days ago


















    7












    $begingroup$

    I think the following anecdote fits well in this category. Note however, that other participants may have experienced these things differently, since they will have had a better background knowledge of the topic.



    At a conference in Uppsala in September 2012, Geordie Williamson was scheduled to give a talk. I can unfortunately not recall the precise topic, as I can no loner find the program for the conference.



    He starts his talk by apologizing that he is in fact going to talk about a completely different topic, since he had very recently finished some work on this with his collaborator Ben Elias.

    He then goes on to describe Soergel's conjecture and some of the ideas that he and Ben have been working on, hoping to make progress on the conjecture.



    The talk is quite technical, involving a lot of quite deep ideas and descriptions of how certain geometrical ideas, such as Hodge theory, can be given more algebraic analogues and how these may be put together to make progress on the conjecture.



    As is typical of any technical talk, it is very hard to keep track of all the details and how they fit together along the way, so he provides a nice summary in the end:



    "In conclusion, Soergel's conjecture is true".






    share|cite|improve this answer











    $endgroup$








    • 3




      $begingroup$
      Well isn’t he lucky that nobody asked him any questions to make him run out of time. To students on MathOverflow: please don’t plan talks like this (with the result at the very end).
      $endgroup$
      – Zach Teitler
      17 hours ago







    • 1




      $begingroup$
      The paper is arxiv.org/abs/1212.0791
      $endgroup$
      – Matt F.
      17 hours ago






    • 4




      $begingroup$
      @ZachTeitler While I agree that it is usually not a good idea to save the main result until the end, going over time should not be an issue for an experienced speaker with an eye on the time and the ability to adapt the talk on the fly, especially when the statement of the main result takes 10 seconds.
      $endgroup$
      – Tobias Kildetoft
      16 hours ago


















    4












    $begingroup$

    Onsager announced in 1948 that he and Kaufman had found a proof for the fact that the spontaneous magnetization of the Ising model on the square lattice with couplings $J_1$ and $J_2$ is given by



    $M = left(1 - left[sinh (2beta J_1) sinh (2beta J_2)right]^-2right)^frac18$



    But he kept the proof a secret as a challenge to the physics community. The proof was obtained by Yang in 1951






    share|cite|improve this answer











    $endgroup$








    • 12




      $begingroup$
      Why do you say “he” when there were two authors? Why do you say “kept the proof a secret” rather than “considered the argument too messy and unrigorous to publish”?
      $endgroup$
      – Matt F.
      2 days ago









    protected by Lucia yesterday



    Thank you for your interest in this question.
    Because it has attracted low-quality or spam answers that had to be removed, posting an answer now requires 10 reputation on this site (the association bonus does not count).



    Would you like to answer one of these unanswered questions instead?














    11 Answers
    11






    active

    oldest

    votes








    11 Answers
    11






    active

    oldest

    votes









    active

    oldest

    votes






    active

    oldest

    votes









    69












    $begingroup$

    The best known lower bound for the minimal length of superpermutations was originally posted anonymously to 4chan.



    The story is told at Mystery Math Whiz and Novelist Advance Permutation Problem, and a publication with a cleaned-up version of the proof is at A lower bound on the length of the shortest superpattern, with "Anonymous 4chan Poster" as the first author. The original 4chan source is archived here.






    share|cite|improve this answer











    $endgroup$








    • 15




      $begingroup$
      Also: a new superpermutation of 7 symbols, shorter than any that was known at the time (8907 symbols long), was posted as a pseudonymous comment on YouTube in February 2019.
      $endgroup$
      – Robin Houston
      Mar 10 at 22:05







    • 24




      $begingroup$
      "Mainly devoted to anime" is a rather kind way to put it.. ;-)
      $endgroup$
      – R..
      Mar 10 at 22:29






    • 18




      $begingroup$
      @R.., as I understand it, the 4chan poster answered a question in a forum dedicated to a particular anime program. The anime in question was meant to be non-linear, and watched in any order. The question was effectively "what is the most efficient way to watch all $n$ episodes of the anime serially, in any order." So it was answered in a forum really "devoted to anime," rather than the average 4chan forum.
      $endgroup$
      – Mark S
      Mar 10 at 23:32







    • 6




      $begingroup$
      @MarkS More specifically, Suzumiya Haruhi no Yuuutsu, and hence the problem was also named "The Haruhi Problem".
      $endgroup$
      – Pedro A
      Mar 11 at 2:04






    • 7




      $begingroup$
      It’s far too late for me to edit my first comment, but I should say that the superpermutation posted as a YouTube comment is 5907 symbols long, not 8907. Apologies for mistyping, and for not noticing sooner.
      $endgroup$
      – Robin Houston
      yesterday















    69












    $begingroup$

    The best known lower bound for the minimal length of superpermutations was originally posted anonymously to 4chan.



    The story is told at Mystery Math Whiz and Novelist Advance Permutation Problem, and a publication with a cleaned-up version of the proof is at A lower bound on the length of the shortest superpattern, with "Anonymous 4chan Poster" as the first author. The original 4chan source is archived here.






    share|cite|improve this answer











    $endgroup$








    • 15




      $begingroup$
      Also: a new superpermutation of 7 symbols, shorter than any that was known at the time (8907 symbols long), was posted as a pseudonymous comment on YouTube in February 2019.
      $endgroup$
      – Robin Houston
      Mar 10 at 22:05







    • 24




      $begingroup$
      "Mainly devoted to anime" is a rather kind way to put it.. ;-)
      $endgroup$
      – R..
      Mar 10 at 22:29






    • 18




      $begingroup$
      @R.., as I understand it, the 4chan poster answered a question in a forum dedicated to a particular anime program. The anime in question was meant to be non-linear, and watched in any order. The question was effectively "what is the most efficient way to watch all $n$ episodes of the anime serially, in any order." So it was answered in a forum really "devoted to anime," rather than the average 4chan forum.
      $endgroup$
      – Mark S
      Mar 10 at 23:32







    • 6




      $begingroup$
      @MarkS More specifically, Suzumiya Haruhi no Yuuutsu, and hence the problem was also named "The Haruhi Problem".
      $endgroup$
      – Pedro A
      Mar 11 at 2:04






    • 7




      $begingroup$
      It’s far too late for me to edit my first comment, but I should say that the superpermutation posted as a YouTube comment is 5907 symbols long, not 8907. Apologies for mistyping, and for not noticing sooner.
      $endgroup$
      – Robin Houston
      yesterday













    69












    69








    69





    $begingroup$

    The best known lower bound for the minimal length of superpermutations was originally posted anonymously to 4chan.



    The story is told at Mystery Math Whiz and Novelist Advance Permutation Problem, and a publication with a cleaned-up version of the proof is at A lower bound on the length of the shortest superpattern, with "Anonymous 4chan Poster" as the first author. The original 4chan source is archived here.






    share|cite|improve this answer











    $endgroup$



    The best known lower bound for the minimal length of superpermutations was originally posted anonymously to 4chan.



    The story is told at Mystery Math Whiz and Novelist Advance Permutation Problem, and a publication with a cleaned-up version of the proof is at A lower bound on the length of the shortest superpattern, with "Anonymous 4chan Poster" as the first author. The original 4chan source is archived here.







    share|cite|improve this answer














    share|cite|improve this answer



    share|cite|improve this answer








    edited 2 days ago


























    community wiki





    3 revs, 2 users 71%
    Carlo Beenakker








    • 15




      $begingroup$
      Also: a new superpermutation of 7 symbols, shorter than any that was known at the time (8907 symbols long), was posted as a pseudonymous comment on YouTube in February 2019.
      $endgroup$
      – Robin Houston
      Mar 10 at 22:05







    • 24




      $begingroup$
      "Mainly devoted to anime" is a rather kind way to put it.. ;-)
      $endgroup$
      – R..
      Mar 10 at 22:29






    • 18




      $begingroup$
      @R.., as I understand it, the 4chan poster answered a question in a forum dedicated to a particular anime program. The anime in question was meant to be non-linear, and watched in any order. The question was effectively "what is the most efficient way to watch all $n$ episodes of the anime serially, in any order." So it was answered in a forum really "devoted to anime," rather than the average 4chan forum.
      $endgroup$
      – Mark S
      Mar 10 at 23:32







    • 6




      $begingroup$
      @MarkS More specifically, Suzumiya Haruhi no Yuuutsu, and hence the problem was also named "The Haruhi Problem".
      $endgroup$
      – Pedro A
      Mar 11 at 2:04






    • 7




      $begingroup$
      It’s far too late for me to edit my first comment, but I should say that the superpermutation posted as a YouTube comment is 5907 symbols long, not 8907. Apologies for mistyping, and for not noticing sooner.
      $endgroup$
      – Robin Houston
      yesterday












    • 15




      $begingroup$
      Also: a new superpermutation of 7 symbols, shorter than any that was known at the time (8907 symbols long), was posted as a pseudonymous comment on YouTube in February 2019.
      $endgroup$
      – Robin Houston
      Mar 10 at 22:05







    • 24




      $begingroup$
      "Mainly devoted to anime" is a rather kind way to put it.. ;-)
      $endgroup$
      – R..
      Mar 10 at 22:29






    • 18




      $begingroup$
      @R.., as I understand it, the 4chan poster answered a question in a forum dedicated to a particular anime program. The anime in question was meant to be non-linear, and watched in any order. The question was effectively "what is the most efficient way to watch all $n$ episodes of the anime serially, in any order." So it was answered in a forum really "devoted to anime," rather than the average 4chan forum.
      $endgroup$
      – Mark S
      Mar 10 at 23:32







    • 6




      $begingroup$
      @MarkS More specifically, Suzumiya Haruhi no Yuuutsu, and hence the problem was also named "The Haruhi Problem".
      $endgroup$
      – Pedro A
      Mar 11 at 2:04






    • 7




      $begingroup$
      It’s far too late for me to edit my first comment, but I should say that the superpermutation posted as a YouTube comment is 5907 symbols long, not 8907. Apologies for mistyping, and for not noticing sooner.
      $endgroup$
      – Robin Houston
      yesterday







    15




    15




    $begingroup$
    Also: a new superpermutation of 7 symbols, shorter than any that was known at the time (8907 symbols long), was posted as a pseudonymous comment on YouTube in February 2019.
    $endgroup$
    – Robin Houston
    Mar 10 at 22:05





    $begingroup$
    Also: a new superpermutation of 7 symbols, shorter than any that was known at the time (8907 symbols long), was posted as a pseudonymous comment on YouTube in February 2019.
    $endgroup$
    – Robin Houston
    Mar 10 at 22:05





    24




    24




    $begingroup$
    "Mainly devoted to anime" is a rather kind way to put it.. ;-)
    $endgroup$
    – R..
    Mar 10 at 22:29




    $begingroup$
    "Mainly devoted to anime" is a rather kind way to put it.. ;-)
    $endgroup$
    – R..
    Mar 10 at 22:29




    18




    18




    $begingroup$
    @R.., as I understand it, the 4chan poster answered a question in a forum dedicated to a particular anime program. The anime in question was meant to be non-linear, and watched in any order. The question was effectively "what is the most efficient way to watch all $n$ episodes of the anime serially, in any order." So it was answered in a forum really "devoted to anime," rather than the average 4chan forum.
    $endgroup$
    – Mark S
    Mar 10 at 23:32





    $begingroup$
    @R.., as I understand it, the 4chan poster answered a question in a forum dedicated to a particular anime program. The anime in question was meant to be non-linear, and watched in any order. The question was effectively "what is the most efficient way to watch all $n$ episodes of the anime serially, in any order." So it was answered in a forum really "devoted to anime," rather than the average 4chan forum.
    $endgroup$
    – Mark S
    Mar 10 at 23:32





    6




    6




    $begingroup$
    @MarkS More specifically, Suzumiya Haruhi no Yuuutsu, and hence the problem was also named "The Haruhi Problem".
    $endgroup$
    – Pedro A
    Mar 11 at 2:04




    $begingroup$
    @MarkS More specifically, Suzumiya Haruhi no Yuuutsu, and hence the problem was also named "The Haruhi Problem".
    $endgroup$
    – Pedro A
    Mar 11 at 2:04




    7




    7




    $begingroup$
    It’s far too late for me to edit my first comment, but I should say that the superpermutation posted as a YouTube comment is 5907 symbols long, not 8907. Apologies for mistyping, and for not noticing sooner.
    $endgroup$
    – Robin Houston
    yesterday




    $begingroup$
    It’s far too late for me to edit my first comment, but I should say that the superpermutation posted as a YouTube comment is 5907 symbols long, not 8907. Apologies for mistyping, and for not noticing sooner.
    $endgroup$
    – Robin Houston
    yesterday











    44












    $begingroup$

    I'll just let below (famous) 1966 article from the Bulletin of the American Mathematical Society speak for itself...



    Lander and Parkin, BAMS 1966




    COUNTEREXAMPLE TO EULER'S CONJECTURE ON SUMS OF LIKE POWERS

    BY L. J. LANDER AND T. R. PARKIN

    Communicated by J. D. Swift, June 27, 1966



    A direct search on the CDC 6600 yielded



    $$ 27^5 + 84^5 + 110^5 + 133^5 = 144^5 $$



    as the smallest instance in which four fifth powers sum to a fifth power. This is a counterexample to a conjecture by Euler [1] that at least $ n $ $ n $th powers are required to sum to an $ n $th power, $ n > 2 $.



    REFERENCE

    1. L. E. Dickson, History of the theory of numbers, Vol. 2, Chelsea, New York, 1952, p. 648.







    share|cite|improve this answer











    $endgroup$

















      44












      $begingroup$

      I'll just let below (famous) 1966 article from the Bulletin of the American Mathematical Society speak for itself...



      Lander and Parkin, BAMS 1966




      COUNTEREXAMPLE TO EULER'S CONJECTURE ON SUMS OF LIKE POWERS

      BY L. J. LANDER AND T. R. PARKIN

      Communicated by J. D. Swift, June 27, 1966



      A direct search on the CDC 6600 yielded



      $$ 27^5 + 84^5 + 110^5 + 133^5 = 144^5 $$



      as the smallest instance in which four fifth powers sum to a fifth power. This is a counterexample to a conjecture by Euler [1] that at least $ n $ $ n $th powers are required to sum to an $ n $th power, $ n > 2 $.



      REFERENCE

      1. L. E. Dickson, History of the theory of numbers, Vol. 2, Chelsea, New York, 1952, p. 648.







      share|cite|improve this answer











      $endgroup$















        44












        44








        44





        $begingroup$

        I'll just let below (famous) 1966 article from the Bulletin of the American Mathematical Society speak for itself...



        Lander and Parkin, BAMS 1966




        COUNTEREXAMPLE TO EULER'S CONJECTURE ON SUMS OF LIKE POWERS

        BY L. J. LANDER AND T. R. PARKIN

        Communicated by J. D. Swift, June 27, 1966



        A direct search on the CDC 6600 yielded



        $$ 27^5 + 84^5 + 110^5 + 133^5 = 144^5 $$



        as the smallest instance in which four fifth powers sum to a fifth power. This is a counterexample to a conjecture by Euler [1] that at least $ n $ $ n $th powers are required to sum to an $ n $th power, $ n > 2 $.



        REFERENCE

        1. L. E. Dickson, History of the theory of numbers, Vol. 2, Chelsea, New York, 1952, p. 648.







        share|cite|improve this answer











        $endgroup$



        I'll just let below (famous) 1966 article from the Bulletin of the American Mathematical Society speak for itself...



        Lander and Parkin, BAMS 1966




        COUNTEREXAMPLE TO EULER'S CONJECTURE ON SUMS OF LIKE POWERS

        BY L. J. LANDER AND T. R. PARKIN

        Communicated by J. D. Swift, June 27, 1966



        A direct search on the CDC 6600 yielded



        $$ 27^5 + 84^5 + 110^5 + 133^5 = 144^5 $$



        as the smallest instance in which four fifth powers sum to a fifth power. This is a counterexample to a conjecture by Euler [1] that at least $ n $ $ n $th powers are required to sum to an $ n $th power, $ n > 2 $.



        REFERENCE

        1. L. E. Dickson, History of the theory of numbers, Vol. 2, Chelsea, New York, 1952, p. 648.








        share|cite|improve this answer














        share|cite|improve this answer



        share|cite|improve this answer








        edited 20 hours ago


























        community wiki





        4 revs, 4 users 55%
        Zsbán Ambrus






















            40












            $begingroup$

            Perelman solving the Poincare "conjecture," posting it only on the arXiv, leaving math, and refusing the Clay prize could be interpreted as a kind of "mic drop."






            share|cite|improve this answer











            $endgroup$








            • 13




              $begingroup$
              Let us not mince words: " 'I'm not interested in money or fame,' he is quoted to have said at the time. 'I don't want to be on display like an animal in a zoo. I'm not a hero of mathematics. I'm not even that successful; that is why I don't want to have everybody looking at me.' "
              $endgroup$
              – Samantha Y
              Mar 11 at 2:40






            • 6




              $begingroup$
              The announcement was posting on arxiv (with no buzzword). Refusing prizes occurred several years later hence is not part of the announcement.
              $endgroup$
              – YCor
              2 days ago






            • 7




              $begingroup$
              @SamanthaY Ironically, I think he's succumbed to the Streisand Effect by doing that. A significant fraction of my awareness of Perelman and the results he's credited with is a result of his refusals to be recognized.
              $endgroup$
              – zibadawa timmy
              2 days ago






            • 6




              $begingroup$
              @zibadawatimmy Yes, but Perelman refused to be owned by the public, which is quite a different thing than wanting privacy/anonymity. To that end, I feel he succeeded.
              $endgroup$
              – Samantha Y
              2 days ago






            • 4




              $begingroup$
              @ycor I apologize :( this entire thread strikes me as unproductive, but that doesn't excuse my taking offense at you, especially as we seem to be in agreement. That is (if I may say so), perpetuating mythologizing the Perelman story, or characterising it as a 'mic drop' will only drive a wedge between mathematicians and the general public, by portraying us as either prize-driven or "odd-balls", and even if somewhere between these two, as drama obsessed gossips. After all, (and this is a very general comment) don't we want everyone to be mathematician? Let them see we're human.
              $endgroup$
              – Samantha Y
              2 days ago
















            40












            $begingroup$

            Perelman solving the Poincare "conjecture," posting it only on the arXiv, leaving math, and refusing the Clay prize could be interpreted as a kind of "mic drop."






            share|cite|improve this answer











            $endgroup$








            • 13




              $begingroup$
              Let us not mince words: " 'I'm not interested in money or fame,' he is quoted to have said at the time. 'I don't want to be on display like an animal in a zoo. I'm not a hero of mathematics. I'm not even that successful; that is why I don't want to have everybody looking at me.' "
              $endgroup$
              – Samantha Y
              Mar 11 at 2:40






            • 6




              $begingroup$
              The announcement was posting on arxiv (with no buzzword). Refusing prizes occurred several years later hence is not part of the announcement.
              $endgroup$
              – YCor
              2 days ago






            • 7




              $begingroup$
              @SamanthaY Ironically, I think he's succumbed to the Streisand Effect by doing that. A significant fraction of my awareness of Perelman and the results he's credited with is a result of his refusals to be recognized.
              $endgroup$
              – zibadawa timmy
              2 days ago






            • 6




              $begingroup$
              @zibadawatimmy Yes, but Perelman refused to be owned by the public, which is quite a different thing than wanting privacy/anonymity. To that end, I feel he succeeded.
              $endgroup$
              – Samantha Y
              2 days ago






            • 4




              $begingroup$
              @ycor I apologize :( this entire thread strikes me as unproductive, but that doesn't excuse my taking offense at you, especially as we seem to be in agreement. That is (if I may say so), perpetuating mythologizing the Perelman story, or characterising it as a 'mic drop' will only drive a wedge between mathematicians and the general public, by portraying us as either prize-driven or "odd-balls", and even if somewhere between these two, as drama obsessed gossips. After all, (and this is a very general comment) don't we want everyone to be mathematician? Let them see we're human.
              $endgroup$
              – Samantha Y
              2 days ago














            40












            40








            40





            $begingroup$

            Perelman solving the Poincare "conjecture," posting it only on the arXiv, leaving math, and refusing the Clay prize could be interpreted as a kind of "mic drop."






            share|cite|improve this answer











            $endgroup$



            Perelman solving the Poincare "conjecture," posting it only on the arXiv, leaving math, and refusing the Clay prize could be interpreted as a kind of "mic drop."







            share|cite|improve this answer














            share|cite|improve this answer



            share|cite|improve this answer








            answered Mar 11 at 1:08


























            community wiki





            Kimball








            • 13




              $begingroup$
              Let us not mince words: " 'I'm not interested in money or fame,' he is quoted to have said at the time. 'I don't want to be on display like an animal in a zoo. I'm not a hero of mathematics. I'm not even that successful; that is why I don't want to have everybody looking at me.' "
              $endgroup$
              – Samantha Y
              Mar 11 at 2:40






            • 6




              $begingroup$
              The announcement was posting on arxiv (with no buzzword). Refusing prizes occurred several years later hence is not part of the announcement.
              $endgroup$
              – YCor
              2 days ago






            • 7




              $begingroup$
              @SamanthaY Ironically, I think he's succumbed to the Streisand Effect by doing that. A significant fraction of my awareness of Perelman and the results he's credited with is a result of his refusals to be recognized.
              $endgroup$
              – zibadawa timmy
              2 days ago






            • 6




              $begingroup$
              @zibadawatimmy Yes, but Perelman refused to be owned by the public, which is quite a different thing than wanting privacy/anonymity. To that end, I feel he succeeded.
              $endgroup$
              – Samantha Y
              2 days ago






            • 4




              $begingroup$
              @ycor I apologize :( this entire thread strikes me as unproductive, but that doesn't excuse my taking offense at you, especially as we seem to be in agreement. That is (if I may say so), perpetuating mythologizing the Perelman story, or characterising it as a 'mic drop' will only drive a wedge between mathematicians and the general public, by portraying us as either prize-driven or "odd-balls", and even if somewhere between these two, as drama obsessed gossips. After all, (and this is a very general comment) don't we want everyone to be mathematician? Let them see we're human.
              $endgroup$
              – Samantha Y
              2 days ago













            • 13




              $begingroup$
              Let us not mince words: " 'I'm not interested in money or fame,' he is quoted to have said at the time. 'I don't want to be on display like an animal in a zoo. I'm not a hero of mathematics. I'm not even that successful; that is why I don't want to have everybody looking at me.' "
              $endgroup$
              – Samantha Y
              Mar 11 at 2:40






            • 6




              $begingroup$
              The announcement was posting on arxiv (with no buzzword). Refusing prizes occurred several years later hence is not part of the announcement.
              $endgroup$
              – YCor
              2 days ago






            • 7




              $begingroup$
              @SamanthaY Ironically, I think he's succumbed to the Streisand Effect by doing that. A significant fraction of my awareness of Perelman and the results he's credited with is a result of his refusals to be recognized.
              $endgroup$
              – zibadawa timmy
              2 days ago






            • 6




              $begingroup$
              @zibadawatimmy Yes, but Perelman refused to be owned by the public, which is quite a different thing than wanting privacy/anonymity. To that end, I feel he succeeded.
              $endgroup$
              – Samantha Y
              2 days ago






            • 4




              $begingroup$
              @ycor I apologize :( this entire thread strikes me as unproductive, but that doesn't excuse my taking offense at you, especially as we seem to be in agreement. That is (if I may say so), perpetuating mythologizing the Perelman story, or characterising it as a 'mic drop' will only drive a wedge between mathematicians and the general public, by portraying us as either prize-driven or "odd-balls", and even if somewhere between these two, as drama obsessed gossips. After all, (and this is a very general comment) don't we want everyone to be mathematician? Let them see we're human.
              $endgroup$
              – Samantha Y
              2 days ago








            13




            13




            $begingroup$
            Let us not mince words: " 'I'm not interested in money or fame,' he is quoted to have said at the time. 'I don't want to be on display like an animal in a zoo. I'm not a hero of mathematics. I'm not even that successful; that is why I don't want to have everybody looking at me.' "
            $endgroup$
            – Samantha Y
            Mar 11 at 2:40




            $begingroup$
            Let us not mince words: " 'I'm not interested in money or fame,' he is quoted to have said at the time. 'I don't want to be on display like an animal in a zoo. I'm not a hero of mathematics. I'm not even that successful; that is why I don't want to have everybody looking at me.' "
            $endgroup$
            – Samantha Y
            Mar 11 at 2:40




            6




            6




            $begingroup$
            The announcement was posting on arxiv (with no buzzword). Refusing prizes occurred several years later hence is not part of the announcement.
            $endgroup$
            – YCor
            2 days ago




            $begingroup$
            The announcement was posting on arxiv (with no buzzword). Refusing prizes occurred several years later hence is not part of the announcement.
            $endgroup$
            – YCor
            2 days ago




            7




            7




            $begingroup$
            @SamanthaY Ironically, I think he's succumbed to the Streisand Effect by doing that. A significant fraction of my awareness of Perelman and the results he's credited with is a result of his refusals to be recognized.
            $endgroup$
            – zibadawa timmy
            2 days ago




            $begingroup$
            @SamanthaY Ironically, I think he's succumbed to the Streisand Effect by doing that. A significant fraction of my awareness of Perelman and the results he's credited with is a result of his refusals to be recognized.
            $endgroup$
            – zibadawa timmy
            2 days ago




            6




            6




            $begingroup$
            @zibadawatimmy Yes, but Perelman refused to be owned by the public, which is quite a different thing than wanting privacy/anonymity. To that end, I feel he succeeded.
            $endgroup$
            – Samantha Y
            2 days ago




            $begingroup$
            @zibadawatimmy Yes, but Perelman refused to be owned by the public, which is quite a different thing than wanting privacy/anonymity. To that end, I feel he succeeded.
            $endgroup$
            – Samantha Y
            2 days ago




            4




            4




            $begingroup$
            @ycor I apologize :( this entire thread strikes me as unproductive, but that doesn't excuse my taking offense at you, especially as we seem to be in agreement. That is (if I may say so), perpetuating mythologizing the Perelman story, or characterising it as a 'mic drop' will only drive a wedge between mathematicians and the general public, by portraying us as either prize-driven or "odd-balls", and even if somewhere between these two, as drama obsessed gossips. After all, (and this is a very general comment) don't we want everyone to be mathematician? Let them see we're human.
            $endgroup$
            – Samantha Y
            2 days ago





            $begingroup$
            @ycor I apologize :( this entire thread strikes me as unproductive, but that doesn't excuse my taking offense at you, especially as we seem to be in agreement. That is (if I may say so), perpetuating mythologizing the Perelman story, or characterising it as a 'mic drop' will only drive a wedge between mathematicians and the general public, by portraying us as either prize-driven or "odd-balls", and even if somewhere between these two, as drama obsessed gossips. After all, (and this is a very general comment) don't we want everyone to be mathematician? Let them see we're human.
            $endgroup$
            – Samantha Y
            2 days ago












            29












            $begingroup$

            Not math but in physics the statistical interpretation of the wave-function was announced by Max Born in a footnote.



            From his paper Zur Quantenmechanik der Stoßvorgänge,




            (1) Anmerkung bei der Korrektur: Genauere Überlegung zeigt, daß die
            Wahrscheinlichkeit dem Quadrat der Größe $Phi_n_tau m$ proportional ist.




            This can be translated as




            (1) Addition in proof: More careful consideration shows that the probability is proportional to the square
            of the quantity $Phi_n_tau m.$




            Because of its implications this is probably the most important footnote in the history of physics. Max Born was awarded the Nobel prize "for his fundamental research in quantum mechanics, especially for his statistical interpretation of the wavefunction".






            share|cite|improve this answer











            $endgroup$








            • 5




              $begingroup$
              I think "Anmerkung bei der Korrektur" is better translated as "Remark added in proof". In particular, it would be a remark by the author, not by the editor. Also, "zeigt" is present tense, "shows" not "will show".
              $endgroup$
              – Andreas Blass
              Mar 11 at 1:44






            • 14




              $begingroup$
              The footnote is not the announcement of a probabilistic interpretation, but a correction that the probability is proportional to $Phi^2$ rather than $Phi$. Also the paper is not so much understated as preliminary, as indicated right below the title.
              $endgroup$
              – Matt F.
              Mar 11 at 2:41











            • $begingroup$
              @AndreasBlass you're right. You're welcome to provide a better translation than the one I found online. If I remember correctly Born added that footnote once the paper was already in the review process
              $endgroup$
              – lcv
              Mar 11 at 3:14






            • 2




              $begingroup$
              @MattF. I take it that your comment means that he didn't mean to show understatement with the footnote. I totally agree. This is however, how the statistical interpretation was brought to public attention. De facto so to speak.
              $endgroup$
              – lcv
              2 days ago















            29












            $begingroup$

            Not math but in physics the statistical interpretation of the wave-function was announced by Max Born in a footnote.



            From his paper Zur Quantenmechanik der Stoßvorgänge,




            (1) Anmerkung bei der Korrektur: Genauere Überlegung zeigt, daß die
            Wahrscheinlichkeit dem Quadrat der Größe $Phi_n_tau m$ proportional ist.




            This can be translated as




            (1) Addition in proof: More careful consideration shows that the probability is proportional to the square
            of the quantity $Phi_n_tau m.$




            Because of its implications this is probably the most important footnote in the history of physics. Max Born was awarded the Nobel prize "for his fundamental research in quantum mechanics, especially for his statistical interpretation of the wavefunction".






            share|cite|improve this answer











            $endgroup$








            • 5




              $begingroup$
              I think "Anmerkung bei der Korrektur" is better translated as "Remark added in proof". In particular, it would be a remark by the author, not by the editor. Also, "zeigt" is present tense, "shows" not "will show".
              $endgroup$
              – Andreas Blass
              Mar 11 at 1:44






            • 14




              $begingroup$
              The footnote is not the announcement of a probabilistic interpretation, but a correction that the probability is proportional to $Phi^2$ rather than $Phi$. Also the paper is not so much understated as preliminary, as indicated right below the title.
              $endgroup$
              – Matt F.
              Mar 11 at 2:41











            • $begingroup$
              @AndreasBlass you're right. You're welcome to provide a better translation than the one I found online. If I remember correctly Born added that footnote once the paper was already in the review process
              $endgroup$
              – lcv
              Mar 11 at 3:14






            • 2




              $begingroup$
              @MattF. I take it that your comment means that he didn't mean to show understatement with the footnote. I totally agree. This is however, how the statistical interpretation was brought to public attention. De facto so to speak.
              $endgroup$
              – lcv
              2 days ago













            29












            29








            29





            $begingroup$

            Not math but in physics the statistical interpretation of the wave-function was announced by Max Born in a footnote.



            From his paper Zur Quantenmechanik der Stoßvorgänge,




            (1) Anmerkung bei der Korrektur: Genauere Überlegung zeigt, daß die
            Wahrscheinlichkeit dem Quadrat der Größe $Phi_n_tau m$ proportional ist.




            This can be translated as




            (1) Addition in proof: More careful consideration shows that the probability is proportional to the square
            of the quantity $Phi_n_tau m.$




            Because of its implications this is probably the most important footnote in the history of physics. Max Born was awarded the Nobel prize "for his fundamental research in quantum mechanics, especially for his statistical interpretation of the wavefunction".






            share|cite|improve this answer











            $endgroup$



            Not math but in physics the statistical interpretation of the wave-function was announced by Max Born in a footnote.



            From his paper Zur Quantenmechanik der Stoßvorgänge,




            (1) Anmerkung bei der Korrektur: Genauere Überlegung zeigt, daß die
            Wahrscheinlichkeit dem Quadrat der Größe $Phi_n_tau m$ proportional ist.




            This can be translated as




            (1) Addition in proof: More careful consideration shows that the probability is proportional to the square
            of the quantity $Phi_n_tau m.$




            Because of its implications this is probably the most important footnote in the history of physics. Max Born was awarded the Nobel prize "for his fundamental research in quantum mechanics, especially for his statistical interpretation of the wavefunction".







            share|cite|improve this answer














            share|cite|improve this answer



            share|cite|improve this answer








            edited Mar 11 at 2:48


























            community wiki





            4 revs, 3 users 77%
            lcv








            • 5




              $begingroup$
              I think "Anmerkung bei der Korrektur" is better translated as "Remark added in proof". In particular, it would be a remark by the author, not by the editor. Also, "zeigt" is present tense, "shows" not "will show".
              $endgroup$
              – Andreas Blass
              Mar 11 at 1:44






            • 14




              $begingroup$
              The footnote is not the announcement of a probabilistic interpretation, but a correction that the probability is proportional to $Phi^2$ rather than $Phi$. Also the paper is not so much understated as preliminary, as indicated right below the title.
              $endgroup$
              – Matt F.
              Mar 11 at 2:41











            • $begingroup$
              @AndreasBlass you're right. You're welcome to provide a better translation than the one I found online. If I remember correctly Born added that footnote once the paper was already in the review process
              $endgroup$
              – lcv
              Mar 11 at 3:14






            • 2




              $begingroup$
              @MattF. I take it that your comment means that he didn't mean to show understatement with the footnote. I totally agree. This is however, how the statistical interpretation was brought to public attention. De facto so to speak.
              $endgroup$
              – lcv
              2 days ago












            • 5




              $begingroup$
              I think "Anmerkung bei der Korrektur" is better translated as "Remark added in proof". In particular, it would be a remark by the author, not by the editor. Also, "zeigt" is present tense, "shows" not "will show".
              $endgroup$
              – Andreas Blass
              Mar 11 at 1:44






            • 14




              $begingroup$
              The footnote is not the announcement of a probabilistic interpretation, but a correction that the probability is proportional to $Phi^2$ rather than $Phi$. Also the paper is not so much understated as preliminary, as indicated right below the title.
              $endgroup$
              – Matt F.
              Mar 11 at 2:41











            • $begingroup$
              @AndreasBlass you're right. You're welcome to provide a better translation than the one I found online. If I remember correctly Born added that footnote once the paper was already in the review process
              $endgroup$
              – lcv
              Mar 11 at 3:14






            • 2




              $begingroup$
              @MattF. I take it that your comment means that he didn't mean to show understatement with the footnote. I totally agree. This is however, how the statistical interpretation was brought to public attention. De facto so to speak.
              $endgroup$
              – lcv
              2 days ago







            5




            5




            $begingroup$
            I think "Anmerkung bei der Korrektur" is better translated as "Remark added in proof". In particular, it would be a remark by the author, not by the editor. Also, "zeigt" is present tense, "shows" not "will show".
            $endgroup$
            – Andreas Blass
            Mar 11 at 1:44




            $begingroup$
            I think "Anmerkung bei der Korrektur" is better translated as "Remark added in proof". In particular, it would be a remark by the author, not by the editor. Also, "zeigt" is present tense, "shows" not "will show".
            $endgroup$
            – Andreas Blass
            Mar 11 at 1:44




            14




            14




            $begingroup$
            The footnote is not the announcement of a probabilistic interpretation, but a correction that the probability is proportional to $Phi^2$ rather than $Phi$. Also the paper is not so much understated as preliminary, as indicated right below the title.
            $endgroup$
            – Matt F.
            Mar 11 at 2:41





            $begingroup$
            The footnote is not the announcement of a probabilistic interpretation, but a correction that the probability is proportional to $Phi^2$ rather than $Phi$. Also the paper is not so much understated as preliminary, as indicated right below the title.
            $endgroup$
            – Matt F.
            Mar 11 at 2:41













            $begingroup$
            @AndreasBlass you're right. You're welcome to provide a better translation than the one I found online. If I remember correctly Born added that footnote once the paper was already in the review process
            $endgroup$
            – lcv
            Mar 11 at 3:14




            $begingroup$
            @AndreasBlass you're right. You're welcome to provide a better translation than the one I found online. If I remember correctly Born added that footnote once the paper was already in the review process
            $endgroup$
            – lcv
            Mar 11 at 3:14




            2




            2




            $begingroup$
            @MattF. I take it that your comment means that he didn't mean to show understatement with the footnote. I totally agree. This is however, how the statistical interpretation was brought to public attention. De facto so to speak.
            $endgroup$
            – lcv
            2 days ago




            $begingroup$
            @MattF. I take it that your comment means that he didn't mean to show understatement with the footnote. I totally agree. This is however, how the statistical interpretation was brought to public attention. De facto so to speak.
            $endgroup$
            – lcv
            2 days ago











            28












            $begingroup$

            Applications of algebra to a problem in topology (YouTube) at Atiyah80 was a talk by Mike Hopkins. In it he announced the solution to the Kervaire invariant one problem in all but one dimension (arXiv, Annals).






            share|cite|improve this answer











            $endgroup$








            • 26




              $begingroup$
              I was in the audience as a second year graduate student, and for me it was just another lecture where I could only understand the first 10-15 minutes. A stranger sitting next to me was very excited afterward, but I remember wondering if he was a nut. So yeah, this qualifies as a mic drop in my book.
              $endgroup$
              – Paul Siegel
              2 days ago










            • $begingroup$
              Can I ask a question about the video? At the beginning there is some discussion of various $pi_n(S^0)$, in particular Pontryagin first errornously claiming that $pi_2(S^0) = 0$ and later realizing that in fact $pi_2(S^0) = mathbbZ/2mathbbZ$. So here is my question: isn't $S^0$ just two points? What would a representative of the non-zero class of $pi_2(S^0)$ look like? You can't get both points of $S^0$ in the range of the map, can you? What is going on here?
              $endgroup$
              – Vincent
              20 hours ago






            • 2




              $begingroup$
              @Vincent from memory he's most likely discussing stable homotopy groups, so that really it's $pi_n+2(S^n)$ for large enough $n$ (in this case, $ngeq 4$), but being sloppy with notation. It would be better denoted $pi_2^s(S^0)$. See eg the pink diagonal in this table starting at $pi_6(S^4)$.
              $endgroup$
              – David Roberts
              20 hours ago







            • 1




              $begingroup$
              @Vincent actually, it's saying the same thing, but I think Hopkins would phrase it as looking at the homotopy groups of the sphere spectrum $mathbbS$, which is cooked up out of $S^0$. Then $pi_2(mathbbS)$ is indeed as he describes.
              $endgroup$
              – David Roberts
              19 hours ago










            • $begingroup$
              @DavidRoberts Thanks!
              $endgroup$
              – Vincent
              19 hours ago















            28












            $begingroup$

            Applications of algebra to a problem in topology (YouTube) at Atiyah80 was a talk by Mike Hopkins. In it he announced the solution to the Kervaire invariant one problem in all but one dimension (arXiv, Annals).






            share|cite|improve this answer











            $endgroup$








            • 26




              $begingroup$
              I was in the audience as a second year graduate student, and for me it was just another lecture where I could only understand the first 10-15 minutes. A stranger sitting next to me was very excited afterward, but I remember wondering if he was a nut. So yeah, this qualifies as a mic drop in my book.
              $endgroup$
              – Paul Siegel
              2 days ago










            • $begingroup$
              Can I ask a question about the video? At the beginning there is some discussion of various $pi_n(S^0)$, in particular Pontryagin first errornously claiming that $pi_2(S^0) = 0$ and later realizing that in fact $pi_2(S^0) = mathbbZ/2mathbbZ$. So here is my question: isn't $S^0$ just two points? What would a representative of the non-zero class of $pi_2(S^0)$ look like? You can't get both points of $S^0$ in the range of the map, can you? What is going on here?
              $endgroup$
              – Vincent
              20 hours ago






            • 2




              $begingroup$
              @Vincent from memory he's most likely discussing stable homotopy groups, so that really it's $pi_n+2(S^n)$ for large enough $n$ (in this case, $ngeq 4$), but being sloppy with notation. It would be better denoted $pi_2^s(S^0)$. See eg the pink diagonal in this table starting at $pi_6(S^4)$.
              $endgroup$
              – David Roberts
              20 hours ago







            • 1




              $begingroup$
              @Vincent actually, it's saying the same thing, but I think Hopkins would phrase it as looking at the homotopy groups of the sphere spectrum $mathbbS$, which is cooked up out of $S^0$. Then $pi_2(mathbbS)$ is indeed as he describes.
              $endgroup$
              – David Roberts
              19 hours ago










            • $begingroup$
              @DavidRoberts Thanks!
              $endgroup$
              – Vincent
              19 hours ago













            28












            28








            28





            $begingroup$

            Applications of algebra to a problem in topology (YouTube) at Atiyah80 was a talk by Mike Hopkins. In it he announced the solution to the Kervaire invariant one problem in all but one dimension (arXiv, Annals).






            share|cite|improve this answer











            $endgroup$



            Applications of algebra to a problem in topology (YouTube) at Atiyah80 was a talk by Mike Hopkins. In it he announced the solution to the Kervaire invariant one problem in all but one dimension (arXiv, Annals).







            share|cite|improve this answer














            share|cite|improve this answer



            share|cite|improve this answer








            answered Mar 10 at 22:48


























            community wiki





            David Roberts








            • 26




              $begingroup$
              I was in the audience as a second year graduate student, and for me it was just another lecture where I could only understand the first 10-15 minutes. A stranger sitting next to me was very excited afterward, but I remember wondering if he was a nut. So yeah, this qualifies as a mic drop in my book.
              $endgroup$
              – Paul Siegel
              2 days ago










            • $begingroup$
              Can I ask a question about the video? At the beginning there is some discussion of various $pi_n(S^0)$, in particular Pontryagin first errornously claiming that $pi_2(S^0) = 0$ and later realizing that in fact $pi_2(S^0) = mathbbZ/2mathbbZ$. So here is my question: isn't $S^0$ just two points? What would a representative of the non-zero class of $pi_2(S^0)$ look like? You can't get both points of $S^0$ in the range of the map, can you? What is going on here?
              $endgroup$
              – Vincent
              20 hours ago






            • 2




              $begingroup$
              @Vincent from memory he's most likely discussing stable homotopy groups, so that really it's $pi_n+2(S^n)$ for large enough $n$ (in this case, $ngeq 4$), but being sloppy with notation. It would be better denoted $pi_2^s(S^0)$. See eg the pink diagonal in this table starting at $pi_6(S^4)$.
              $endgroup$
              – David Roberts
              20 hours ago







            • 1




              $begingroup$
              @Vincent actually, it's saying the same thing, but I think Hopkins would phrase it as looking at the homotopy groups of the sphere spectrum $mathbbS$, which is cooked up out of $S^0$. Then $pi_2(mathbbS)$ is indeed as he describes.
              $endgroup$
              – David Roberts
              19 hours ago










            • $begingroup$
              @DavidRoberts Thanks!
              $endgroup$
              – Vincent
              19 hours ago












            • 26




              $begingroup$
              I was in the audience as a second year graduate student, and for me it was just another lecture where I could only understand the first 10-15 minutes. A stranger sitting next to me was very excited afterward, but I remember wondering if he was a nut. So yeah, this qualifies as a mic drop in my book.
              $endgroup$
              – Paul Siegel
              2 days ago










            • $begingroup$
              Can I ask a question about the video? At the beginning there is some discussion of various $pi_n(S^0)$, in particular Pontryagin first errornously claiming that $pi_2(S^0) = 0$ and later realizing that in fact $pi_2(S^0) = mathbbZ/2mathbbZ$. So here is my question: isn't $S^0$ just two points? What would a representative of the non-zero class of $pi_2(S^0)$ look like? You can't get both points of $S^0$ in the range of the map, can you? What is going on here?
              $endgroup$
              – Vincent
              20 hours ago






            • 2




              $begingroup$
              @Vincent from memory he's most likely discussing stable homotopy groups, so that really it's $pi_n+2(S^n)$ for large enough $n$ (in this case, $ngeq 4$), but being sloppy with notation. It would be better denoted $pi_2^s(S^0)$. See eg the pink diagonal in this table starting at $pi_6(S^4)$.
              $endgroup$
              – David Roberts
              20 hours ago







            • 1




              $begingroup$
              @Vincent actually, it's saying the same thing, but I think Hopkins would phrase it as looking at the homotopy groups of the sphere spectrum $mathbbS$, which is cooked up out of $S^0$. Then $pi_2(mathbbS)$ is indeed as he describes.
              $endgroup$
              – David Roberts
              19 hours ago










            • $begingroup$
              @DavidRoberts Thanks!
              $endgroup$
              – Vincent
              19 hours ago







            26




            26




            $begingroup$
            I was in the audience as a second year graduate student, and for me it was just another lecture where I could only understand the first 10-15 minutes. A stranger sitting next to me was very excited afterward, but I remember wondering if he was a nut. So yeah, this qualifies as a mic drop in my book.
            $endgroup$
            – Paul Siegel
            2 days ago




            $begingroup$
            I was in the audience as a second year graduate student, and for me it was just another lecture where I could only understand the first 10-15 minutes. A stranger sitting next to me was very excited afterward, but I remember wondering if he was a nut. So yeah, this qualifies as a mic drop in my book.
            $endgroup$
            – Paul Siegel
            2 days ago












            $begingroup$
            Can I ask a question about the video? At the beginning there is some discussion of various $pi_n(S^0)$, in particular Pontryagin first errornously claiming that $pi_2(S^0) = 0$ and later realizing that in fact $pi_2(S^0) = mathbbZ/2mathbbZ$. So here is my question: isn't $S^0$ just two points? What would a representative of the non-zero class of $pi_2(S^0)$ look like? You can't get both points of $S^0$ in the range of the map, can you? What is going on here?
            $endgroup$
            – Vincent
            20 hours ago




            $begingroup$
            Can I ask a question about the video? At the beginning there is some discussion of various $pi_n(S^0)$, in particular Pontryagin first errornously claiming that $pi_2(S^0) = 0$ and later realizing that in fact $pi_2(S^0) = mathbbZ/2mathbbZ$. So here is my question: isn't $S^0$ just two points? What would a representative of the non-zero class of $pi_2(S^0)$ look like? You can't get both points of $S^0$ in the range of the map, can you? What is going on here?
            $endgroup$
            – Vincent
            20 hours ago




            2




            2




            $begingroup$
            @Vincent from memory he's most likely discussing stable homotopy groups, so that really it's $pi_n+2(S^n)$ for large enough $n$ (in this case, $ngeq 4$), but being sloppy with notation. It would be better denoted $pi_2^s(S^0)$. See eg the pink diagonal in this table starting at $pi_6(S^4)$.
            $endgroup$
            – David Roberts
            20 hours ago





            $begingroup$
            @Vincent from memory he's most likely discussing stable homotopy groups, so that really it's $pi_n+2(S^n)$ for large enough $n$ (in this case, $ngeq 4$), but being sloppy with notation. It would be better denoted $pi_2^s(S^0)$. See eg the pink diagonal in this table starting at $pi_6(S^4)$.
            $endgroup$
            – David Roberts
            20 hours ago





            1




            1




            $begingroup$
            @Vincent actually, it's saying the same thing, but I think Hopkins would phrase it as looking at the homotopy groups of the sphere spectrum $mathbbS$, which is cooked up out of $S^0$. Then $pi_2(mathbbS)$ is indeed as he describes.
            $endgroup$
            – David Roberts
            19 hours ago




            $begingroup$
            @Vincent actually, it's saying the same thing, but I think Hopkins would phrase it as looking at the homotopy groups of the sphere spectrum $mathbbS$, which is cooked up out of $S^0$. Then $pi_2(mathbbS)$ is indeed as he describes.
            $endgroup$
            – David Roberts
            19 hours ago












            $begingroup$
            @DavidRoberts Thanks!
            $endgroup$
            – Vincent
            19 hours ago




            $begingroup$
            @DavidRoberts Thanks!
            $endgroup$
            – Vincent
            19 hours ago











            23












            $begingroup$

            I consider this manner as a mark of a professional mathematician: let others convey the excitement of a discovery. A good recent example was the submission of a paper on bounded gaps between primes. Much of the public excitement was generated by people other than the author, Yitang Zhang.



            Gerhard "Can Be Excited In Private" Paseman, 2019.03.10.






            share|cite|improve this answer











            $endgroup$








            • 3




              $begingroup$
              I especially like his understated comment that "I believe one could make it sharper" when asked if he thought $k<70,000,000$ could be reduced.
              $endgroup$
              – Mark S
              Mar 10 at 23:17






            • 5




              $begingroup$
              Well, a distinction can be drawn between the most professional approach, which I guess is to submit the work to the Annals or another top journal, accept invitations to speak about it, etc. followed by Yitang Zhang and the more dramatic (and fun) approach where you post it only to your personal website, refuse to tell people what your talk announcing the result is about in advance, leave math immediately afterwards, etc. It seems that the "mic drop" refers to examples that go above and beyond what you'd do for a usual strong result.
              $endgroup$
              – Will Sawin
              Mar 11 at 1:34






            • 1




              $begingroup$
              And then there is of course Grothendieck who let Borel & Serre publish his proof of the GRR theorem...
              $endgroup$
              – Einfacher Schreiberling
              yesterday















            23












            $begingroup$

            I consider this manner as a mark of a professional mathematician: let others convey the excitement of a discovery. A good recent example was the submission of a paper on bounded gaps between primes. Much of the public excitement was generated by people other than the author, Yitang Zhang.



            Gerhard "Can Be Excited In Private" Paseman, 2019.03.10.






            share|cite|improve this answer











            $endgroup$








            • 3




              $begingroup$
              I especially like his understated comment that "I believe one could make it sharper" when asked if he thought $k<70,000,000$ could be reduced.
              $endgroup$
              – Mark S
              Mar 10 at 23:17






            • 5




              $begingroup$
              Well, a distinction can be drawn between the most professional approach, which I guess is to submit the work to the Annals or another top journal, accept invitations to speak about it, etc. followed by Yitang Zhang and the more dramatic (and fun) approach where you post it only to your personal website, refuse to tell people what your talk announcing the result is about in advance, leave math immediately afterwards, etc. It seems that the "mic drop" refers to examples that go above and beyond what you'd do for a usual strong result.
              $endgroup$
              – Will Sawin
              Mar 11 at 1:34






            • 1




              $begingroup$
              And then there is of course Grothendieck who let Borel & Serre publish his proof of the GRR theorem...
              $endgroup$
              – Einfacher Schreiberling
              yesterday













            23












            23








            23





            $begingroup$

            I consider this manner as a mark of a professional mathematician: let others convey the excitement of a discovery. A good recent example was the submission of a paper on bounded gaps between primes. Much of the public excitement was generated by people other than the author, Yitang Zhang.



            Gerhard "Can Be Excited In Private" Paseman, 2019.03.10.






            share|cite|improve this answer











            $endgroup$



            I consider this manner as a mark of a professional mathematician: let others convey the excitement of a discovery. A good recent example was the submission of a paper on bounded gaps between primes. Much of the public excitement was generated by people other than the author, Yitang Zhang.



            Gerhard "Can Be Excited In Private" Paseman, 2019.03.10.







            share|cite|improve this answer














            share|cite|improve this answer



            share|cite|improve this answer








            answered Mar 10 at 21:06


























            community wiki





            Gerhard Paseman








            • 3




              $begingroup$
              I especially like his understated comment that "I believe one could make it sharper" when asked if he thought $k<70,000,000$ could be reduced.
              $endgroup$
              – Mark S
              Mar 10 at 23:17






            • 5




              $begingroup$
              Well, a distinction can be drawn between the most professional approach, which I guess is to submit the work to the Annals or another top journal, accept invitations to speak about it, etc. followed by Yitang Zhang and the more dramatic (and fun) approach where you post it only to your personal website, refuse to tell people what your talk announcing the result is about in advance, leave math immediately afterwards, etc. It seems that the "mic drop" refers to examples that go above and beyond what you'd do for a usual strong result.
              $endgroup$
              – Will Sawin
              Mar 11 at 1:34






            • 1




              $begingroup$
              And then there is of course Grothendieck who let Borel & Serre publish his proof of the GRR theorem...
              $endgroup$
              – Einfacher Schreiberling
              yesterday












            • 3




              $begingroup$
              I especially like his understated comment that "I believe one could make it sharper" when asked if he thought $k<70,000,000$ could be reduced.
              $endgroup$
              – Mark S
              Mar 10 at 23:17






            • 5




              $begingroup$
              Well, a distinction can be drawn between the most professional approach, which I guess is to submit the work to the Annals or another top journal, accept invitations to speak about it, etc. followed by Yitang Zhang and the more dramatic (and fun) approach where you post it only to your personal website, refuse to tell people what your talk announcing the result is about in advance, leave math immediately afterwards, etc. It seems that the "mic drop" refers to examples that go above and beyond what you'd do for a usual strong result.
              $endgroup$
              – Will Sawin
              Mar 11 at 1:34






            • 1




              $begingroup$
              And then there is of course Grothendieck who let Borel & Serre publish his proof of the GRR theorem...
              $endgroup$
              – Einfacher Schreiberling
              yesterday







            3




            3




            $begingroup$
            I especially like his understated comment that "I believe one could make it sharper" when asked if he thought $k<70,000,000$ could be reduced.
            $endgroup$
            – Mark S
            Mar 10 at 23:17




            $begingroup$
            I especially like his understated comment that "I believe one could make it sharper" when asked if he thought $k<70,000,000$ could be reduced.
            $endgroup$
            – Mark S
            Mar 10 at 23:17




            5




            5




            $begingroup$
            Well, a distinction can be drawn between the most professional approach, which I guess is to submit the work to the Annals or another top journal, accept invitations to speak about it, etc. followed by Yitang Zhang and the more dramatic (and fun) approach where you post it only to your personal website, refuse to tell people what your talk announcing the result is about in advance, leave math immediately afterwards, etc. It seems that the "mic drop" refers to examples that go above and beyond what you'd do for a usual strong result.
            $endgroup$
            – Will Sawin
            Mar 11 at 1:34




            $begingroup$
            Well, a distinction can be drawn between the most professional approach, which I guess is to submit the work to the Annals or another top journal, accept invitations to speak about it, etc. followed by Yitang Zhang and the more dramatic (and fun) approach where you post it only to your personal website, refuse to tell people what your talk announcing the result is about in advance, leave math immediately afterwards, etc. It seems that the "mic drop" refers to examples that go above and beyond what you'd do for a usual strong result.
            $endgroup$
            – Will Sawin
            Mar 11 at 1:34




            1




            1




            $begingroup$
            And then there is of course Grothendieck who let Borel & Serre publish his proof of the GRR theorem...
            $endgroup$
            – Einfacher Schreiberling
            yesterday




            $begingroup$
            And then there is of course Grothendieck who let Borel & Serre publish his proof of the GRR theorem...
            $endgroup$
            – Einfacher Schreiberling
            yesterday











            22












            $begingroup$

            From the Wikipedia article on Frank Nelson Cole:




            On October 31, 1903, Cole famously made a presentation to a meeting of
            the American Mathematical Society where he identified the factors of
            the Mersenne number $2^67$ − 1, or M67.[5] Édouard Lucas had demonstrated
            in 1876 that M67 must have factors (i.e., is not prime), but he was
            unable to determine what those factors were. During Cole's so-called
            "lecture", he approached the chalkboard and in complete silence
            proceeded to calculate the value of M67, with the result being
            147,573,952,589,676,412,927. Cole then moved to the other side of the
            board and wrote 193,707,721 × 761,838,257,287, and worked through the
            tedious calculations by hand. Upon completing the multiplication and
            demonstrating that the result equaled M67, Cole returned to his seat,
            not having uttered a word during the hour-long presentation. His
            audience greeted the presentation with a standing ovation.







            share|cite|improve this answer











            $endgroup$








            • 13




              $begingroup$
              I'm interested in the historiography of this urban legend. Is the only source for the above E. T. Bell? If so, must it be considered suspect, because E. T. Bell was a much better mythmaker than a biographer? I'd like to believe it to be true - a broken clock is still right twice a day...
              $endgroup$
              – Mark S
              Mar 11 at 0:41






            • 12




              $begingroup$
              Maybe things were different in 1903, but I would not give a standing ovation for an hour of silent arithmetic. Also I’m sorry but those calculations don’t seem like they would take an hour. None of it seems believable. Still a fun story though.
              $endgroup$
              – Zach Teitler
              Mar 11 at 1:18






            • 3




              $begingroup$
              @ZachTeitler Maybe $M_67$ was a really big deal in 1903? Maybe actually finding the factors was generally greeted with some expression of acclamation? Mersenne antedates Fermat by a dozen or so years, $M_67$ was effectively open for just as long in 1903 as FLT was. I'm pretty sure that people stood up and clapped at the end of Wiles' lecture in 1993. Of course Wiles' lecture was not an "hour of silent arithmetic," so maybe that part is a stretch.
              $endgroup$
              – Mark S
              Mar 11 at 2:57







            • 5




              $begingroup$
              $M_67$ would be a big deal any time and finding those factors would have certainly been worthy of acclaim. I just meant that there would be far better ways to present the factorization than grinding through the arithmetic. As an audience member I would be far, far more interested in how Cole found those factors, than in whether he remembered to carry the $3$ or whatever. An hour of that would have been tough to sit through. Although... maybe at one of those 20-minute AMS special sessions, perhaps.... :-)
              $endgroup$
              – Zach Teitler
              Mar 11 at 5:32






            • 3




              $begingroup$
              @ZachTeitler right, so the "applause/standing ovation" is not so suspect, but the "silent lecture of multiplication" seems bogus and inconsistent with even mathematicians of 1903...
              $endgroup$
              – Mark S
              2 days ago















            22












            $begingroup$

            From the Wikipedia article on Frank Nelson Cole:




            On October 31, 1903, Cole famously made a presentation to a meeting of
            the American Mathematical Society where he identified the factors of
            the Mersenne number $2^67$ − 1, or M67.[5] Édouard Lucas had demonstrated
            in 1876 that M67 must have factors (i.e., is not prime), but he was
            unable to determine what those factors were. During Cole's so-called
            "lecture", he approached the chalkboard and in complete silence
            proceeded to calculate the value of M67, with the result being
            147,573,952,589,676,412,927. Cole then moved to the other side of the
            board and wrote 193,707,721 × 761,838,257,287, and worked through the
            tedious calculations by hand. Upon completing the multiplication and
            demonstrating that the result equaled M67, Cole returned to his seat,
            not having uttered a word during the hour-long presentation. His
            audience greeted the presentation with a standing ovation.







            share|cite|improve this answer











            $endgroup$








            • 13




              $begingroup$
              I'm interested in the historiography of this urban legend. Is the only source for the above E. T. Bell? If so, must it be considered suspect, because E. T. Bell was a much better mythmaker than a biographer? I'd like to believe it to be true - a broken clock is still right twice a day...
              $endgroup$
              – Mark S
              Mar 11 at 0:41






            • 12




              $begingroup$
              Maybe things were different in 1903, but I would not give a standing ovation for an hour of silent arithmetic. Also I’m sorry but those calculations don’t seem like they would take an hour. None of it seems believable. Still a fun story though.
              $endgroup$
              – Zach Teitler
              Mar 11 at 1:18






            • 3




              $begingroup$
              @ZachTeitler Maybe $M_67$ was a really big deal in 1903? Maybe actually finding the factors was generally greeted with some expression of acclamation? Mersenne antedates Fermat by a dozen or so years, $M_67$ was effectively open for just as long in 1903 as FLT was. I'm pretty sure that people stood up and clapped at the end of Wiles' lecture in 1993. Of course Wiles' lecture was not an "hour of silent arithmetic," so maybe that part is a stretch.
              $endgroup$
              – Mark S
              Mar 11 at 2:57







            • 5




              $begingroup$
              $M_67$ would be a big deal any time and finding those factors would have certainly been worthy of acclaim. I just meant that there would be far better ways to present the factorization than grinding through the arithmetic. As an audience member I would be far, far more interested in how Cole found those factors, than in whether he remembered to carry the $3$ or whatever. An hour of that would have been tough to sit through. Although... maybe at one of those 20-minute AMS special sessions, perhaps.... :-)
              $endgroup$
              – Zach Teitler
              Mar 11 at 5:32






            • 3




              $begingroup$
              @ZachTeitler right, so the "applause/standing ovation" is not so suspect, but the "silent lecture of multiplication" seems bogus and inconsistent with even mathematicians of 1903...
              $endgroup$
              – Mark S
              2 days ago













            22












            22








            22





            $begingroup$

            From the Wikipedia article on Frank Nelson Cole:




            On October 31, 1903, Cole famously made a presentation to a meeting of
            the American Mathematical Society where he identified the factors of
            the Mersenne number $2^67$ − 1, or M67.[5] Édouard Lucas had demonstrated
            in 1876 that M67 must have factors (i.e., is not prime), but he was
            unable to determine what those factors were. During Cole's so-called
            "lecture", he approached the chalkboard and in complete silence
            proceeded to calculate the value of M67, with the result being
            147,573,952,589,676,412,927. Cole then moved to the other side of the
            board and wrote 193,707,721 × 761,838,257,287, and worked through the
            tedious calculations by hand. Upon completing the multiplication and
            demonstrating that the result equaled M67, Cole returned to his seat,
            not having uttered a word during the hour-long presentation. His
            audience greeted the presentation with a standing ovation.







            share|cite|improve this answer











            $endgroup$



            From the Wikipedia article on Frank Nelson Cole:




            On October 31, 1903, Cole famously made a presentation to a meeting of
            the American Mathematical Society where he identified the factors of
            the Mersenne number $2^67$ − 1, or M67.[5] Édouard Lucas had demonstrated
            in 1876 that M67 must have factors (i.e., is not prime), but he was
            unable to determine what those factors were. During Cole's so-called
            "lecture", he approached the chalkboard and in complete silence
            proceeded to calculate the value of M67, with the result being
            147,573,952,589,676,412,927. Cole then moved to the other side of the
            board and wrote 193,707,721 × 761,838,257,287, and worked through the
            tedious calculations by hand. Upon completing the multiplication and
            demonstrating that the result equaled M67, Cole returned to his seat,
            not having uttered a word during the hour-long presentation. His
            audience greeted the presentation with a standing ovation.








            share|cite|improve this answer














            share|cite|improve this answer



            share|cite|improve this answer








            edited Mar 11 at 1:23


























            community wiki





            2 revs, 2 users 97%
            Jeff Strom








            • 13




              $begingroup$
              I'm interested in the historiography of this urban legend. Is the only source for the above E. T. Bell? If so, must it be considered suspect, because E. T. Bell was a much better mythmaker than a biographer? I'd like to believe it to be true - a broken clock is still right twice a day...
              $endgroup$
              – Mark S
              Mar 11 at 0:41






            • 12




              $begingroup$
              Maybe things were different in 1903, but I would not give a standing ovation for an hour of silent arithmetic. Also I’m sorry but those calculations don’t seem like they would take an hour. None of it seems believable. Still a fun story though.
              $endgroup$
              – Zach Teitler
              Mar 11 at 1:18






            • 3




              $begingroup$
              @ZachTeitler Maybe $M_67$ was a really big deal in 1903? Maybe actually finding the factors was generally greeted with some expression of acclamation? Mersenne antedates Fermat by a dozen or so years, $M_67$ was effectively open for just as long in 1903 as FLT was. I'm pretty sure that people stood up and clapped at the end of Wiles' lecture in 1993. Of course Wiles' lecture was not an "hour of silent arithmetic," so maybe that part is a stretch.
              $endgroup$
              – Mark S
              Mar 11 at 2:57







            • 5




              $begingroup$
              $M_67$ would be a big deal any time and finding those factors would have certainly been worthy of acclaim. I just meant that there would be far better ways to present the factorization than grinding through the arithmetic. As an audience member I would be far, far more interested in how Cole found those factors, than in whether he remembered to carry the $3$ or whatever. An hour of that would have been tough to sit through. Although... maybe at one of those 20-minute AMS special sessions, perhaps.... :-)
              $endgroup$
              – Zach Teitler
              Mar 11 at 5:32






            • 3




              $begingroup$
              @ZachTeitler right, so the "applause/standing ovation" is not so suspect, but the "silent lecture of multiplication" seems bogus and inconsistent with even mathematicians of 1903...
              $endgroup$
              – Mark S
              2 days ago












            • 13




              $begingroup$
              I'm interested in the historiography of this urban legend. Is the only source for the above E. T. Bell? If so, must it be considered suspect, because E. T. Bell was a much better mythmaker than a biographer? I'd like to believe it to be true - a broken clock is still right twice a day...
              $endgroup$
              – Mark S
              Mar 11 at 0:41






            • 12




              $begingroup$
              Maybe things were different in 1903, but I would not give a standing ovation for an hour of silent arithmetic. Also I’m sorry but those calculations don’t seem like they would take an hour. None of it seems believable. Still a fun story though.
              $endgroup$
              – Zach Teitler
              Mar 11 at 1:18






            • 3




              $begingroup$
              @ZachTeitler Maybe $M_67$ was a really big deal in 1903? Maybe actually finding the factors was generally greeted with some expression of acclamation? Mersenne antedates Fermat by a dozen or so years, $M_67$ was effectively open for just as long in 1903 as FLT was. I'm pretty sure that people stood up and clapped at the end of Wiles' lecture in 1993. Of course Wiles' lecture was not an "hour of silent arithmetic," so maybe that part is a stretch.
              $endgroup$
              – Mark S
              Mar 11 at 2:57







            • 5




              $begingroup$
              $M_67$ would be a big deal any time and finding those factors would have certainly been worthy of acclaim. I just meant that there would be far better ways to present the factorization than grinding through the arithmetic. As an audience member I would be far, far more interested in how Cole found those factors, than in whether he remembered to carry the $3$ or whatever. An hour of that would have been tough to sit through. Although... maybe at one of those 20-minute AMS special sessions, perhaps.... :-)
              $endgroup$
              – Zach Teitler
              Mar 11 at 5:32






            • 3




              $begingroup$
              @ZachTeitler right, so the "applause/standing ovation" is not so suspect, but the "silent lecture of multiplication" seems bogus and inconsistent with even mathematicians of 1903...
              $endgroup$
              – Mark S
              2 days ago







            13




            13




            $begingroup$
            I'm interested in the historiography of this urban legend. Is the only source for the above E. T. Bell? If so, must it be considered suspect, because E. T. Bell was a much better mythmaker than a biographer? I'd like to believe it to be true - a broken clock is still right twice a day...
            $endgroup$
            – Mark S
            Mar 11 at 0:41




            $begingroup$
            I'm interested in the historiography of this urban legend. Is the only source for the above E. T. Bell? If so, must it be considered suspect, because E. T. Bell was a much better mythmaker than a biographer? I'd like to believe it to be true - a broken clock is still right twice a day...
            $endgroup$
            – Mark S
            Mar 11 at 0:41




            12




            12




            $begingroup$
            Maybe things were different in 1903, but I would not give a standing ovation for an hour of silent arithmetic. Also I’m sorry but those calculations don’t seem like they would take an hour. None of it seems believable. Still a fun story though.
            $endgroup$
            – Zach Teitler
            Mar 11 at 1:18




            $begingroup$
            Maybe things were different in 1903, but I would not give a standing ovation for an hour of silent arithmetic. Also I’m sorry but those calculations don’t seem like they would take an hour. None of it seems believable. Still a fun story though.
            $endgroup$
            – Zach Teitler
            Mar 11 at 1:18




            3




            3




            $begingroup$
            @ZachTeitler Maybe $M_67$ was a really big deal in 1903? Maybe actually finding the factors was generally greeted with some expression of acclamation? Mersenne antedates Fermat by a dozen or so years, $M_67$ was effectively open for just as long in 1903 as FLT was. I'm pretty sure that people stood up and clapped at the end of Wiles' lecture in 1993. Of course Wiles' lecture was not an "hour of silent arithmetic," so maybe that part is a stretch.
            $endgroup$
            – Mark S
            Mar 11 at 2:57





            $begingroup$
            @ZachTeitler Maybe $M_67$ was a really big deal in 1903? Maybe actually finding the factors was generally greeted with some expression of acclamation? Mersenne antedates Fermat by a dozen or so years, $M_67$ was effectively open for just as long in 1903 as FLT was. I'm pretty sure that people stood up and clapped at the end of Wiles' lecture in 1993. Of course Wiles' lecture was not an "hour of silent arithmetic," so maybe that part is a stretch.
            $endgroup$
            – Mark S
            Mar 11 at 2:57





            5




            5




            $begingroup$
            $M_67$ would be a big deal any time and finding those factors would have certainly been worthy of acclaim. I just meant that there would be far better ways to present the factorization than grinding through the arithmetic. As an audience member I would be far, far more interested in how Cole found those factors, than in whether he remembered to carry the $3$ or whatever. An hour of that would have been tough to sit through. Although... maybe at one of those 20-minute AMS special sessions, perhaps.... :-)
            $endgroup$
            – Zach Teitler
            Mar 11 at 5:32




            $begingroup$
            $M_67$ would be a big deal any time and finding those factors would have certainly been worthy of acclaim. I just meant that there would be far better ways to present the factorization than grinding through the arithmetic. As an audience member I would be far, far more interested in how Cole found those factors, than in whether he remembered to carry the $3$ or whatever. An hour of that would have been tough to sit through. Although... maybe at one of those 20-minute AMS special sessions, perhaps.... :-)
            $endgroup$
            – Zach Teitler
            Mar 11 at 5:32




            3




            3




            $begingroup$
            @ZachTeitler right, so the "applause/standing ovation" is not so suspect, but the "silent lecture of multiplication" seems bogus and inconsistent with even mathematicians of 1903...
            $endgroup$
            – Mark S
            2 days ago




            $begingroup$
            @ZachTeitler right, so the "applause/standing ovation" is not so suspect, but the "silent lecture of multiplication" seems bogus and inconsistent with even mathematicians of 1903...
            $endgroup$
            – Mark S
            2 days ago











            17












            $begingroup$

            Kurt Gödel, only a few days before Hilbert gives his famous "We must know – We will know!" quote, had just proven that we cannot know.



            Namely, any reasonably strong foundation of mathematics, if it has a finitary proof verification process, cannot decide all the true statements. Mathematics, in its essence, is incomplete.



            Philosophically speaking, perhaps one of the biggest mic drop moments. Metaphorically, this virtual coinciding with Hilbert's lecture just makes the room even more silent afterwards.






            share|cite|improve this answer











            $endgroup$








            • 1




              $begingroup$
              Cool story - when was the lecture? Is there any contemporaneous reporting of the audience reaction?
              $endgroup$
              – Mark S
              2 days ago






            • 18




              $begingroup$
              This took place in 1930 in Koenigsberg, which was Hilbert's hometown. Goedel's talk was a day or two before Hilbert's talk, not at the same conference, and Hilbert may not have even been at that talk. For more, see hsm.stackexchange.com/questions/29/… and maa.org/book/export/html/326610.
              $endgroup$
              – KConrad
              2 days ago











            • $begingroup$
              @KConrad: The silent room is a metaphor here. Because the usual reaction to a mic drop is that the room goes silent for a moment. :-)
              $endgroup$
              – Asaf Karagila
              2 days ago






            • 2




              $begingroup$
              Apparently von Neumann, in the audience, remarked at the end of Gödel's lecture "It's all over". Unfortunately I do not have a good source for this.
              $endgroup$
              – David Roberts
              18 hours ago















            17












            $begingroup$

            Kurt Gödel, only a few days before Hilbert gives his famous "We must know – We will know!" quote, had just proven that we cannot know.



            Namely, any reasonably strong foundation of mathematics, if it has a finitary proof verification process, cannot decide all the true statements. Mathematics, in its essence, is incomplete.



            Philosophically speaking, perhaps one of the biggest mic drop moments. Metaphorically, this virtual coinciding with Hilbert's lecture just makes the room even more silent afterwards.






            share|cite|improve this answer











            $endgroup$








            • 1




              $begingroup$
              Cool story - when was the lecture? Is there any contemporaneous reporting of the audience reaction?
              $endgroup$
              – Mark S
              2 days ago






            • 18




              $begingroup$
              This took place in 1930 in Koenigsberg, which was Hilbert's hometown. Goedel's talk was a day or two before Hilbert's talk, not at the same conference, and Hilbert may not have even been at that talk. For more, see hsm.stackexchange.com/questions/29/… and maa.org/book/export/html/326610.
              $endgroup$
              – KConrad
              2 days ago











            • $begingroup$
              @KConrad: The silent room is a metaphor here. Because the usual reaction to a mic drop is that the room goes silent for a moment. :-)
              $endgroup$
              – Asaf Karagila
              2 days ago






            • 2




              $begingroup$
              Apparently von Neumann, in the audience, remarked at the end of Gödel's lecture "It's all over". Unfortunately I do not have a good source for this.
              $endgroup$
              – David Roberts
              18 hours ago













            17












            17








            17





            $begingroup$

            Kurt Gödel, only a few days before Hilbert gives his famous "We must know – We will know!" quote, had just proven that we cannot know.



            Namely, any reasonably strong foundation of mathematics, if it has a finitary proof verification process, cannot decide all the true statements. Mathematics, in its essence, is incomplete.



            Philosophically speaking, perhaps one of the biggest mic drop moments. Metaphorically, this virtual coinciding with Hilbert's lecture just makes the room even more silent afterwards.






            share|cite|improve this answer











            $endgroup$



            Kurt Gödel, only a few days before Hilbert gives his famous "We must know – We will know!" quote, had just proven that we cannot know.



            Namely, any reasonably strong foundation of mathematics, if it has a finitary proof verification process, cannot decide all the true statements. Mathematics, in its essence, is incomplete.



            Philosophically speaking, perhaps one of the biggest mic drop moments. Metaphorically, this virtual coinciding with Hilbert's lecture just makes the room even more silent afterwards.







            share|cite|improve this answer














            share|cite|improve this answer



            share|cite|improve this answer








            edited 14 hours ago


























            community wiki





            2 revs, 2 users 80%
            Asaf Karagila








            • 1




              $begingroup$
              Cool story - when was the lecture? Is there any contemporaneous reporting of the audience reaction?
              $endgroup$
              – Mark S
              2 days ago






            • 18




              $begingroup$
              This took place in 1930 in Koenigsberg, which was Hilbert's hometown. Goedel's talk was a day or two before Hilbert's talk, not at the same conference, and Hilbert may not have even been at that talk. For more, see hsm.stackexchange.com/questions/29/… and maa.org/book/export/html/326610.
              $endgroup$
              – KConrad
              2 days ago











            • $begingroup$
              @KConrad: The silent room is a metaphor here. Because the usual reaction to a mic drop is that the room goes silent for a moment. :-)
              $endgroup$
              – Asaf Karagila
              2 days ago






            • 2




              $begingroup$
              Apparently von Neumann, in the audience, remarked at the end of Gödel's lecture "It's all over". Unfortunately I do not have a good source for this.
              $endgroup$
              – David Roberts
              18 hours ago












            • 1




              $begingroup$
              Cool story - when was the lecture? Is there any contemporaneous reporting of the audience reaction?
              $endgroup$
              – Mark S
              2 days ago






            • 18




              $begingroup$
              This took place in 1930 in Koenigsberg, which was Hilbert's hometown. Goedel's talk was a day or two before Hilbert's talk, not at the same conference, and Hilbert may not have even been at that talk. For more, see hsm.stackexchange.com/questions/29/… and maa.org/book/export/html/326610.
              $endgroup$
              – KConrad
              2 days ago











            • $begingroup$
              @KConrad: The silent room is a metaphor here. Because the usual reaction to a mic drop is that the room goes silent for a moment. :-)
              $endgroup$
              – Asaf Karagila
              2 days ago






            • 2




              $begingroup$
              Apparently von Neumann, in the audience, remarked at the end of Gödel's lecture "It's all over". Unfortunately I do not have a good source for this.
              $endgroup$
              – David Roberts
              18 hours ago







            1




            1




            $begingroup$
            Cool story - when was the lecture? Is there any contemporaneous reporting of the audience reaction?
            $endgroup$
            – Mark S
            2 days ago




            $begingroup$
            Cool story - when was the lecture? Is there any contemporaneous reporting of the audience reaction?
            $endgroup$
            – Mark S
            2 days ago




            18




            18




            $begingroup$
            This took place in 1930 in Koenigsberg, which was Hilbert's hometown. Goedel's talk was a day or two before Hilbert's talk, not at the same conference, and Hilbert may not have even been at that talk. For more, see hsm.stackexchange.com/questions/29/… and maa.org/book/export/html/326610.
            $endgroup$
            – KConrad
            2 days ago





            $begingroup$
            This took place in 1930 in Koenigsberg, which was Hilbert's hometown. Goedel's talk was a day or two before Hilbert's talk, not at the same conference, and Hilbert may not have even been at that talk. For more, see hsm.stackexchange.com/questions/29/… and maa.org/book/export/html/326610.
            $endgroup$
            – KConrad
            2 days ago













            $begingroup$
            @KConrad: The silent room is a metaphor here. Because the usual reaction to a mic drop is that the room goes silent for a moment. :-)
            $endgroup$
            – Asaf Karagila
            2 days ago




            $begingroup$
            @KConrad: The silent room is a metaphor here. Because the usual reaction to a mic drop is that the room goes silent for a moment. :-)
            $endgroup$
            – Asaf Karagila
            2 days ago




            2




            2




            $begingroup$
            Apparently von Neumann, in the audience, remarked at the end of Gödel's lecture "It's all over". Unfortunately I do not have a good source for this.
            $endgroup$
            – David Roberts
            18 hours ago




            $begingroup$
            Apparently von Neumann, in the audience, remarked at the end of Gödel's lecture "It's all over". Unfortunately I do not have a good source for this.
            $endgroup$
            – David Roberts
            18 hours ago











            7












            $begingroup$

            My favorite is non-mathematician Marjorie Rice challenging the proof of "No other pentagon tilings exist" with multiple new pentagon tilings. Schattschneider's article was the primary announcement of the results.






            share|cite|improve this answer











            $endgroup$








            • 6




              $begingroup$
              While an excellent result, how was this a "mic drop"?
              $endgroup$
              – Noah Schweber
              2 days ago















            7












            $begingroup$

            My favorite is non-mathematician Marjorie Rice challenging the proof of "No other pentagon tilings exist" with multiple new pentagon tilings. Schattschneider's article was the primary announcement of the results.






            share|cite|improve this answer











            $endgroup$








            • 6




              $begingroup$
              While an excellent result, how was this a "mic drop"?
              $endgroup$
              – Noah Schweber
              2 days ago













            7












            7








            7





            $begingroup$

            My favorite is non-mathematician Marjorie Rice challenging the proof of "No other pentagon tilings exist" with multiple new pentagon tilings. Schattschneider's article was the primary announcement of the results.






            share|cite|improve this answer











            $endgroup$



            My favorite is non-mathematician Marjorie Rice challenging the proof of "No other pentagon tilings exist" with multiple new pentagon tilings. Schattschneider's article was the primary announcement of the results.







            share|cite|improve this answer














            share|cite|improve this answer



            share|cite|improve this answer








            edited 2 days ago


























            community wiki





            2 revs, 2 users 67%
            Matt F.








            • 6




              $begingroup$
              While an excellent result, how was this a "mic drop"?
              $endgroup$
              – Noah Schweber
              2 days ago












            • 6




              $begingroup$
              While an excellent result, how was this a "mic drop"?
              $endgroup$
              – Noah Schweber
              2 days ago







            6




            6




            $begingroup$
            While an excellent result, how was this a "mic drop"?
            $endgroup$
            – Noah Schweber
            2 days ago




            $begingroup$
            While an excellent result, how was this a "mic drop"?
            $endgroup$
            – Noah Schweber
            2 days ago











            7












            $begingroup$

            I think the following anecdote fits well in this category. Note however, that other participants may have experienced these things differently, since they will have had a better background knowledge of the topic.



            At a conference in Uppsala in September 2012, Geordie Williamson was scheduled to give a talk. I can unfortunately not recall the precise topic, as I can no loner find the program for the conference.



            He starts his talk by apologizing that he is in fact going to talk about a completely different topic, since he had very recently finished some work on this with his collaborator Ben Elias.

            He then goes on to describe Soergel's conjecture and some of the ideas that he and Ben have been working on, hoping to make progress on the conjecture.



            The talk is quite technical, involving a lot of quite deep ideas and descriptions of how certain geometrical ideas, such as Hodge theory, can be given more algebraic analogues and how these may be put together to make progress on the conjecture.



            As is typical of any technical talk, it is very hard to keep track of all the details and how they fit together along the way, so he provides a nice summary in the end:



            "In conclusion, Soergel's conjecture is true".






            share|cite|improve this answer











            $endgroup$








            • 3




              $begingroup$
              Well isn’t he lucky that nobody asked him any questions to make him run out of time. To students on MathOverflow: please don’t plan talks like this (with the result at the very end).
              $endgroup$
              – Zach Teitler
              17 hours ago







            • 1




              $begingroup$
              The paper is arxiv.org/abs/1212.0791
              $endgroup$
              – Matt F.
              17 hours ago






            • 4




              $begingroup$
              @ZachTeitler While I agree that it is usually not a good idea to save the main result until the end, going over time should not be an issue for an experienced speaker with an eye on the time and the ability to adapt the talk on the fly, especially when the statement of the main result takes 10 seconds.
              $endgroup$
              – Tobias Kildetoft
              16 hours ago















            7












            $begingroup$

            I think the following anecdote fits well in this category. Note however, that other participants may have experienced these things differently, since they will have had a better background knowledge of the topic.



            At a conference in Uppsala in September 2012, Geordie Williamson was scheduled to give a talk. I can unfortunately not recall the precise topic, as I can no loner find the program for the conference.



            He starts his talk by apologizing that he is in fact going to talk about a completely different topic, since he had very recently finished some work on this with his collaborator Ben Elias.

            He then goes on to describe Soergel's conjecture and some of the ideas that he and Ben have been working on, hoping to make progress on the conjecture.



            The talk is quite technical, involving a lot of quite deep ideas and descriptions of how certain geometrical ideas, such as Hodge theory, can be given more algebraic analogues and how these may be put together to make progress on the conjecture.



            As is typical of any technical talk, it is very hard to keep track of all the details and how they fit together along the way, so he provides a nice summary in the end:



            "In conclusion, Soergel's conjecture is true".






            share|cite|improve this answer











            $endgroup$








            • 3




              $begingroup$
              Well isn’t he lucky that nobody asked him any questions to make him run out of time. To students on MathOverflow: please don’t plan talks like this (with the result at the very end).
              $endgroup$
              – Zach Teitler
              17 hours ago







            • 1




              $begingroup$
              The paper is arxiv.org/abs/1212.0791
              $endgroup$
              – Matt F.
              17 hours ago






            • 4




              $begingroup$
              @ZachTeitler While I agree that it is usually not a good idea to save the main result until the end, going over time should not be an issue for an experienced speaker with an eye on the time and the ability to adapt the talk on the fly, especially when the statement of the main result takes 10 seconds.
              $endgroup$
              – Tobias Kildetoft
              16 hours ago













            7












            7








            7





            $begingroup$

            I think the following anecdote fits well in this category. Note however, that other participants may have experienced these things differently, since they will have had a better background knowledge of the topic.



            At a conference in Uppsala in September 2012, Geordie Williamson was scheduled to give a talk. I can unfortunately not recall the precise topic, as I can no loner find the program for the conference.



            He starts his talk by apologizing that he is in fact going to talk about a completely different topic, since he had very recently finished some work on this with his collaborator Ben Elias.

            He then goes on to describe Soergel's conjecture and some of the ideas that he and Ben have been working on, hoping to make progress on the conjecture.



            The talk is quite technical, involving a lot of quite deep ideas and descriptions of how certain geometrical ideas, such as Hodge theory, can be given more algebraic analogues and how these may be put together to make progress on the conjecture.



            As is typical of any technical talk, it is very hard to keep track of all the details and how they fit together along the way, so he provides a nice summary in the end:



            "In conclusion, Soergel's conjecture is true".






            share|cite|improve this answer











            $endgroup$



            I think the following anecdote fits well in this category. Note however, that other participants may have experienced these things differently, since they will have had a better background knowledge of the topic.



            At a conference in Uppsala in September 2012, Geordie Williamson was scheduled to give a talk. I can unfortunately not recall the precise topic, as I can no loner find the program for the conference.



            He starts his talk by apologizing that he is in fact going to talk about a completely different topic, since he had very recently finished some work on this with his collaborator Ben Elias.

            He then goes on to describe Soergel's conjecture and some of the ideas that he and Ben have been working on, hoping to make progress on the conjecture.



            The talk is quite technical, involving a lot of quite deep ideas and descriptions of how certain geometrical ideas, such as Hodge theory, can be given more algebraic analogues and how these may be put together to make progress on the conjecture.



            As is typical of any technical talk, it is very hard to keep track of all the details and how they fit together along the way, so he provides a nice summary in the end:



            "In conclusion, Soergel's conjecture is true".







            share|cite|improve this answer














            share|cite|improve this answer



            share|cite|improve this answer








            answered 17 hours ago


























            community wiki





            Tobias Kildetoft








            • 3




              $begingroup$
              Well isn’t he lucky that nobody asked him any questions to make him run out of time. To students on MathOverflow: please don’t plan talks like this (with the result at the very end).
              $endgroup$
              – Zach Teitler
              17 hours ago







            • 1




              $begingroup$
              The paper is arxiv.org/abs/1212.0791
              $endgroup$
              – Matt F.
              17 hours ago






            • 4




              $begingroup$
              @ZachTeitler While I agree that it is usually not a good idea to save the main result until the end, going over time should not be an issue for an experienced speaker with an eye on the time and the ability to adapt the talk on the fly, especially when the statement of the main result takes 10 seconds.
              $endgroup$
              – Tobias Kildetoft
              16 hours ago












            • 3




              $begingroup$
              Well isn’t he lucky that nobody asked him any questions to make him run out of time. To students on MathOverflow: please don’t plan talks like this (with the result at the very end).
              $endgroup$
              – Zach Teitler
              17 hours ago







            • 1




              $begingroup$
              The paper is arxiv.org/abs/1212.0791
              $endgroup$
              – Matt F.
              17 hours ago






            • 4




              $begingroup$
              @ZachTeitler While I agree that it is usually not a good idea to save the main result until the end, going over time should not be an issue for an experienced speaker with an eye on the time and the ability to adapt the talk on the fly, especially when the statement of the main result takes 10 seconds.
              $endgroup$
              – Tobias Kildetoft
              16 hours ago







            3




            3




            $begingroup$
            Well isn’t he lucky that nobody asked him any questions to make him run out of time. To students on MathOverflow: please don’t plan talks like this (with the result at the very end).
            $endgroup$
            – Zach Teitler
            17 hours ago





            $begingroup$
            Well isn’t he lucky that nobody asked him any questions to make him run out of time. To students on MathOverflow: please don’t plan talks like this (with the result at the very end).
            $endgroup$
            – Zach Teitler
            17 hours ago





            1




            1




            $begingroup$
            The paper is arxiv.org/abs/1212.0791
            $endgroup$
            – Matt F.
            17 hours ago




            $begingroup$
            The paper is arxiv.org/abs/1212.0791
            $endgroup$
            – Matt F.
            17 hours ago




            4




            4




            $begingroup$
            @ZachTeitler While I agree that it is usually not a good idea to save the main result until the end, going over time should not be an issue for an experienced speaker with an eye on the time and the ability to adapt the talk on the fly, especially when the statement of the main result takes 10 seconds.
            $endgroup$
            – Tobias Kildetoft
            16 hours ago




            $begingroup$
            @ZachTeitler While I agree that it is usually not a good idea to save the main result until the end, going over time should not be an issue for an experienced speaker with an eye on the time and the ability to adapt the talk on the fly, especially when the statement of the main result takes 10 seconds.
            $endgroup$
            – Tobias Kildetoft
            16 hours ago











            4












            $begingroup$

            Onsager announced in 1948 that he and Kaufman had found a proof for the fact that the spontaneous magnetization of the Ising model on the square lattice with couplings $J_1$ and $J_2$ is given by



            $M = left(1 - left[sinh (2beta J_1) sinh (2beta J_2)right]^-2right)^frac18$



            But he kept the proof a secret as a challenge to the physics community. The proof was obtained by Yang in 1951






            share|cite|improve this answer











            $endgroup$








            • 12




              $begingroup$
              Why do you say “he” when there were two authors? Why do you say “kept the proof a secret” rather than “considered the argument too messy and unrigorous to publish”?
              $endgroup$
              – Matt F.
              2 days ago















            4












            $begingroup$

            Onsager announced in 1948 that he and Kaufman had found a proof for the fact that the spontaneous magnetization of the Ising model on the square lattice with couplings $J_1$ and $J_2$ is given by



            $M = left(1 - left[sinh (2beta J_1) sinh (2beta J_2)right]^-2right)^frac18$



            But he kept the proof a secret as a challenge to the physics community. The proof was obtained by Yang in 1951






            share|cite|improve this answer











            $endgroup$








            • 12




              $begingroup$
              Why do you say “he” when there were two authors? Why do you say “kept the proof a secret” rather than “considered the argument too messy and unrigorous to publish”?
              $endgroup$
              – Matt F.
              2 days ago













            4












            4








            4





            $begingroup$

            Onsager announced in 1948 that he and Kaufman had found a proof for the fact that the spontaneous magnetization of the Ising model on the square lattice with couplings $J_1$ and $J_2$ is given by



            $M = left(1 - left[sinh (2beta J_1) sinh (2beta J_2)right]^-2right)^frac18$



            But he kept the proof a secret as a challenge to the physics community. The proof was obtained by Yang in 1951






            share|cite|improve this answer











            $endgroup$



            Onsager announced in 1948 that he and Kaufman had found a proof for the fact that the spontaneous magnetization of the Ising model on the square lattice with couplings $J_1$ and $J_2$ is given by



            $M = left(1 - left[sinh (2beta J_1) sinh (2beta J_2)right]^-2right)^frac18$



            But he kept the proof a secret as a challenge to the physics community. The proof was obtained by Yang in 1951







            share|cite|improve this answer














            share|cite|improve this answer



            share|cite|improve this answer








            answered Mar 11 at 3:05


























            community wiki





            Count Iblis








            • 12




              $begingroup$
              Why do you say “he” when there were two authors? Why do you say “kept the proof a secret” rather than “considered the argument too messy and unrigorous to publish”?
              $endgroup$
              – Matt F.
              2 days ago












            • 12




              $begingroup$
              Why do you say “he” when there were two authors? Why do you say “kept the proof a secret” rather than “considered the argument too messy and unrigorous to publish”?
              $endgroup$
              – Matt F.
              2 days ago







            12




            12




            $begingroup$
            Why do you say “he” when there were two authors? Why do you say “kept the proof a secret” rather than “considered the argument too messy and unrigorous to publish”?
            $endgroup$
            – Matt F.
            2 days ago




            $begingroup$
            Why do you say “he” when there were two authors? Why do you say “kept the proof a secret” rather than “considered the argument too messy and unrigorous to publish”?
            $endgroup$
            – Matt F.
            2 days ago





            protected by Lucia yesterday



            Thank you for your interest in this question.
            Because it has attracted low-quality or spam answers that had to be removed, posting an answer now requires 10 reputation on this site (the association bonus does not count).



            Would you like to answer one of these unanswered questions instead?



            Popular posts from this blog

            Lowndes Grove History Architecture References Navigation menu32°48′6″N 79°57′58″W / 32.80167°N 79.96611°W / 32.80167; -79.9661132°48′6″N 79°57′58″W / 32.80167°N 79.96611°W / 32.80167; -79.9661178002500"National Register Information System"Historic houses of South Carolina"Lowndes Grove""+32° 48' 6.00", −79° 57' 58.00""Lowndes Grove, Charleston County (260 St. Margaret St., Charleston)""Lowndes Grove"The Charleston ExpositionIt Happened in South Carolina"Lowndes Grove (House), Saint Margaret Street & Sixth Avenue, Charleston, Charleston County, SC(Photographs)"Plantations of the Carolina Low Countrye

            random experiment with two different functions on unit interval Announcing the arrival of Valued Associate #679: Cesar Manara Planned maintenance scheduled April 23, 2019 at 00:00UTC (8:00pm US/Eastern)Random variable and probability space notionsRandom Walk with EdgesFinding functions where the increase over a random interval is Poisson distributedNumber of days until dayCan an observed event in fact be of zero probability?Unit random processmodels of coins and uniform distributionHow to get the number of successes given $n$ trials , probability $P$ and a random variable $X$Absorbing Markov chain in a computer. Is “almost every” turned into always convergence in computer executions?Stopped random walk is not uniformly integrable

            How should I support this large drywall patch? Planned maintenance scheduled April 23, 2019 at 00:00UTC (8:00pm US/Eastern) Announcing the arrival of Valued Associate #679: Cesar Manara Unicorn Meta Zoo #1: Why another podcast?How do I cover large gaps in drywall?How do I keep drywall around a patch from crumbling?Can I glue a second layer of drywall?How to patch long strip on drywall?Large drywall patch: how to avoid bulging seams?Drywall Mesh Patch vs. Bulge? To remove or not to remove?How to fix this drywall job?Prep drywall before backsplashWhat's the best way to fix this horrible drywall patch job?Drywall patching using 3M Patch Plus Primer